You are on page 1of 490
SOLVED PROBLEMS IN a hA(CS S.L. SRIVASTAVA Published by ATLANTIC PUBLISHERS & DISTRIBUTORS B-2, Vishal Enclave, Opp. Rajouri Garden, New Delhi-110027 Phones : 25413460, 25429987, 25466842 Sales Office 7/22, Ansari Road, Darya Ganj, New Delhi-110002 Phones: 23273880, 23275880, 23280451 Fax : 91-11-23285873 web : wwwatlanticbooks.com e-mail : info@atlanticbooks.com Copyright © The Author, 2004 All rights reserved. No part of this publication may be reproduced, stored in a retrieval system, transmitted or utilized in any form or by any means, electronic, mechanical, photocopying, recording or otherwise, without the prior permission of the copyright owner. Application for such permission should be addressed to the publisher. Printed in India at Nice Printing Press, Delhi CONTENTS Preface v Volume 1 1._Kinematics in One and Two Dimensions 2. Particle Dynamics 39 4. Collision and Rigid Body Rotation 184 5. Fluid Pressure and Buoyancy 313 6. Elasticity 333 7. Simple Harmonic Motion 366 8. Surface Tension and Flow of Liquids 466 9. Sound 525 Volume 2 10. Heat 583 11. Electrostatics 71 12. Simple Circuit and Electrolysis 822 13. Electromagnetism 882 14. Electromagnetic Induction and Alternating Current 939 15. Optics 1012 16. Atomic Physics 1142 17. Valve and Semiconductor 1267 1 E AND Two DIMENSIONS KINEMATICS IN 1.1, Aman walks 15 km north and then 14.1 km south-east. What is his resultant displacement from the origin? Solution: Let us represent the displacement along the x axis and ‘ y axis, keeping the x axis to the east and y axis to the north as shown in Fig. 1.1. For the first part of the y journey the displacement is along w is €----x axis the positive side of the y axis and is given y, = 15 km. For the second part of the journey the displacement 14.1 km s has two components. Fig. 1.1 The x component of the displacement is 14.1008 45=14.1% 2. = 14 Lokam v2 141 The y component of the displacement is y axis — 14.1 sin 45 =- 14.1 x 1 -10km v2 Net displacement along x axis = 10 km. Net displacement along y axis = 15-10=5km Resultant displacement = 100+25 =5/5km 1,2. Two objects are approaching a point Oas 2v shown in Fig. 1.2 with equal velocities V. Determine the relative velocity of one with respect to the other. | Solution: In Fig. 1.2 the two velocities are with respect to the earth. If we assume that object 2 is at rest, then the earth moves with a Fig. 1.2 s SOLVED PROBLEMS IN PHYSICS velocity V towards object 2, Now the velocity of object 1 with respect to the earth + velocity of the earth with respect to object 2 will be equal to the velocity of object 1 with respect to object 2, i.e. "Wa Wv Thus the velocity of object 1 with respect to object 2 is equal to difference in their velocities with respect to the earth. The speed is worked out as V,. = WV?+V? = 2V 1.3. An object covers distance AB in the following manner. Half of the distance AB, viz. AB/2 = AC, is covered with velocity v,. Of the remaining half, distance CD is covered with velocity v, for half of the time and DB with velocity v, for the other half of the time. Find the mean velocity of the object. Solution: Let the total distance AB shown in Fig. 1.3 be equal to x. AC will now be equal to x/2. . x Time taken to cover distance AC= —— A c D B 2u, Fi ig. 1.3 cD Time taken to cover distance CD = — Yy . DB Time taken to cover the distance DB = a Di Now in view of the problem, a = — andCD+DB=~ Y% v, 2 Total distance covered = x x ~. Mean velocity = <., CD, DB 4%, YU = - since —— = DB *_, 26D he 2, ° 4 x xy 1 2u, U+v, Y, _ (4 +¥,) VU, +U, +205 aa You have either reached a page that is unavailable for viewing or reached your viewing limit for this book. aa You have either reached a page that is unavailable for viewing or reached your viewing limit for this book. aa You have either reached a page that is unavailable for viewing or reached your viewing limit for this book. 6 SOLVED PROBLEMS IN PHYSICS . VOA—3yF +P total time t= ~~" seme dg oF sah x— a 4 NX2(QA-x)x-1 x)x- | Now ax =0=- 7 sf 2 (OAc xP aE | - Fig. 1.7 or ts 1 (OA - x) xP +P or AC? +P =4?-AC? 1.10. A ship goes from A to B at v, = 10 km/hr and from B to A at v, = 16 km/hr. Find (i) The average velocity of the ship (ii) the velocity of the river current. Solution: (i) Let xbe the distance between A and B. Time for the entire path (up and down journey) 1-2 x _ 13x18x = Toxs * 16x5 ~~ 400 18 18 aay _ 2¥x400 Average velocity = Taxlax = 3.42 m/sec i 5 5 Gi) 10 x je TUT 16x -u 5 2u= 3 u= 3 = 0.83 m/sec 1,11. A boat moves perpendicular to the bank with velocity of 7.2 km/hr (2 m/sec). The current carries it 150 m down stream. Find (i) the velocity of the current (ii) the time required to cross the river. The river is 0.5 km wide. je 150 m1 z u_ 150 Solution: tan@ = 27 500 os “4 = 0.6 m/sec km t= am = 250 sec p aa You have either reached a page that is unavailable for viewing or reached your viewing limit for this book. aa You have either reached a page that is unavailable for viewing or reached your viewing limit for this book. aa You have either reached a page that is unavailable for viewing or reached your viewing limit for this book. aa You have either reached a page that is unavailable for viewing or reached your viewing limit for this book. aa You have either reached a page that is unavailable for viewing or reached your viewing limit for this book. aa You have either reached a page that is unavailable for viewing or reached your viewing limit for this book. aa You have either reached a page that is unavailable for viewing or reached your viewing limit for this book. 14 SOLVED PROBLEMS IN PHYSICS 1.23. A small block of mass m slides down an inclined plane forming an angle a = 30° with the horizontal. The coefficient of friction k between the block and the plane is no longer constant but varies with x, the distance travelled by the block on the plane, according to the equation k=ax, where ais a constant. Find the distance travelled and the maximum velocity attained by the block before coming to rest. Solution: The equation of motion of the x ae block along the plane (see Fig. 1.18) is given by mg sin a. —k mg cos a=m — or gsina-k corn tS s S ~ dx dt Fig. 1.18 . dv [ de _ or gsina-kgcosa=v #|¢- o| or (gsina-gaxcosa) dx=vdv On integrating the above equation, we have 2 vp gsina -x- ~~ gcos a= > This is the equation for the velocity of the block. On settig v = 0 we have x= 2 tanaorx= 2 x a a a 3 4 v For obtaining the maximum value of v, we equate —— to zero ax ie, gsina-agxcosa=— =0 .e., 8 dk tana or x= tana Le. atadistance x= the velocity of the block will be maximum. 1 tana a-g tan’a Sy? epsings: Soe Now 3 Vmax = § Sino a 2 a sina-tana a _ Sane sne 8 sina- tana a 2a cosa & => sina-tana 2a aa You have either reached a page that is unavailable for viewing or reached your viewing limit for this book. aa You have either reached a page that is unavailable for viewing or reached your viewing limit for this book. aa You have either reached a page that is unavailable for viewing or reached your viewing limit for this book. 18 SOLVED PROBLEMS IN PHYSICS 1.29. Two particles are projected in the earths gravitational field from a point with initial velocities v, and v, and at angles 9, and 6, with the horizontal respectively. What will be the path of one particle as seen from the other. Solution: The separation betwen the two particles along x axis (taken as horizontal) is equal to x= Xx,—x, = (v, cos @,—v, cos @,) t Similarly the separation between the particles along y axis is equal to y= (» sino, ¢-@" 2 (» sin@, Jar} y=(u,sin0,-v, sin,)t These separations are in fact the coordinates of a point on one trajectory relative to the other ¥ , ¥sin®, -4sinG _ const =m x v, C080, —v, cos 8, o y=me The path will therefore be a straight line. 1.30. A spherical ball is located at a point A (see Fig. 1.21) at a height h above the earth's surface. As it starts falling freely another similar ball is thrown from the earth’s surface from the point O. With what initial velocity and at what angle should the ball be thrown from O in order to collide with the first one at B at an altitude h, = h/2 after covering a horizontal distance 1? Solution: Time taken by the first ball for the path ABis the same as that of the second ball for the horizontal distance 1. Now ap = 2a] gp 2 2 or ° cvsine. [E_2 ——i—— =vsine- “3 Fig. 1.21 aa You have either reached a page that is unavailable for viewing or reached your viewing limit for this book. aa You have either reached a page that is unavailable for viewing or reached your viewing limit for this book. aa You have either reached a page that is unavailable for viewing or reached your viewing limit for this book. 22 SOLVED PROBLEMS IN PHYSICS uy pat = gcos® a _ = _gno dx’ ~ vicos’6 =0ato ay _-8 dx” ~ vecos’@ v2cos?0 & 1.34. An object is situated on a hill h metres high has an elevation 8. In order to hit this object show that the initial velocity of projection should not be less than /gh (1 + cosec B) . A Solution: Let O be the point from which rl an object is projected with an initial velocity u in order to hit the object positioned at A. In view of Fig. 1.25, we have OA=hcosec B ° Fig. 1.25 8 Now if u is minimum then OA will be the maximum range, i.e., 2 < Ih a. g(1+sin B) or u = Jgh(1+cosecB) 1,35. Two particles A and B starts moving from a high point Oat t=0 in the opposite direction with horizontal velocities of 3 m/s and 4 m/s respectively. Due to earth's gravitational field the two particles experiences uniform acceleration g = 9.8 m/s*. How far apart are the particles when their velocity vectors become mutually perpendicular. Solution: Let us assume that v, = 3 m/s and v, = 4 m/s. In view of the geometry of the hcosec B = OA= Fig. 1.26 we have the following: “ ! S r aya? 6,+0,= 2 Me } ¢G2 1 1 sin 8, = gt Mi 7 / i (gt? +?) fy ! ‘ ' sin0,= Le L (g* +0) ~~ Fig. 1.26 aa You have either reached a page that is unavailable for viewing or reached your viewing limit for this book. aa You have either reached a page that is unavailable for viewing or reached your viewing limit for this book. aa You have either reached a page that is unavailable for viewing or reached your viewing limit for this book. 26 SOLVED PROBLEMS IN PHYSICS h and ¢t,=2 z Now (f,-1,)?=(t, + 1)? -4tt, (t,-¢)? 2. 16h sinro- Bh & & le 1 (t,-1) =| 2 asin? 0- of Further 2-,/gh - cos@ x (t,-f,) =2h or _—e [Ses 6- i} & on (2 sin? 6-1) gcos’ 6 1 = 16 sin? @ cos? 6 - 8 cos’ 6 1 = 16 (1 - cos? 8) cos? 6-8 cos? 6 1 = 16 cos’ 6 - 16 cos* @- 8 cos’? 16 cos‘ 6-8 cos?6+1=0 or (4cos?@-1)?=0 1 ry nes or cos’ G=75 3 nos sin? 6 = 4 With these values wef Bank 1.40. A ball is xiiewea in xy plane with a velocity v, and at an angle 0 with the horizontal. During motion it experiences an air resistance equal to + Bu where f is a positive constant and vis the instantaneous velocity. Find (i) the time taken to attain the highest point (ii) the maximum height attained by the ball. Solution: (i) The equation of motion of the ball along y axis is given by mdv a7 (mg + Bu) aa You have either reached a page that is unavailable for viewing or reached your viewing limit for this book. aa You have either reached a page that is unavailable for viewing or reached your viewing limit for this book. aa You have either reached a page that is unavailable for viewing or reached your viewing limit for this book. aa You have either reached a page that is unavailable for viewing or reached your viewing limit for this book. aa You have either reached a page that is unavailable for viewing or reached your viewing limit for this book. aa You have either reached a page that is unavailable for viewing or reached your viewing limit for this book. aa You have either reached a page that is unavailable for viewing or reached your viewing limit for this book. 34 SOLVED PROBLEMS IN PHYSICS 8 Total distance covered by the disc = fe — Ro) do 3 Ro? sb-— Further, | = RO alt “R On substituting the value of 0 the total distance covered by the disc PoP Pc Time taken by the disc t= Velocity v Length covered On substituting the values of various terms we have = 2Rv = 2x0.25x2 ~*° 1.52. A ball, initially at rest, starts falling down asmooth plane inclined at angle a = 30° with the horizontal. After falling through the dis- tance h= 4m, the ball rebounds elastically off the plane (Fig. 1.36a). At what distance from the point of impact will the ball rebound for the second time? Solution: The ball strikes the inclined plane with a velocity v = J2gh where h is the distance travelled by the ball in air. The initial velocity of the ball at the first impact is v. In view of the Fig. 1.36b, v makes an angle a with the y-axis. Velocity v will have two components, v sin a = v, along the plane and v cos a = v, perpendicular to the plane. Let | be the distance travelled along the plane between the first and second impact. Then, according to the laws of motion, 1 lsvsina-t+ > -gsina-?; gsina is the component of acceleration due to gravity along the plane. Here tis the time of flight. = Fig. 1.36a aa You have either reached a page that is unavailable for viewing or reached your viewing limit for this book. aa You have either reached a page that is unavailable for viewing or reached your viewing limit for this book. aa You have either reached a page that is unavailable for viewing or reached your viewing limit for this book. 38 SOLVED PROBLEMS IN PHYSICS V, = velocity of ball projected from Qat the point O (point ofcollision) = ¥(11.28c0s 45)’ +(11.28 sin 45)" + 2x 105.25 = 63.62 + 63.62 +105 = $232.24 = 15.24 m/s 1.56. Three tortoises are located at the vertices of an equilateral triangle of side a = 3 m. They start moving simultaneously with constant velocity v=1 m/s. The first heads for the second, the second for the third and the third for the first, ie., all the time they move in cyclic order. After what time will they meet? Solution: Let three tortoises be located at the corners of the triangle ABC (Fig. 1.40). Ais moving towards B, Btowards Cand Ctowards A. In triangle 2 a_a ABC, OA = OB= OC= = 3 -~ = = 8 339 v3 The velocity of the tortoise at A can be resolved along AO and perpendicular to AO. Similarly the velocities of the tortoise at B and Ccan be resolved respectively along BO and perpendicular to BO, and CO and perpendicular to CO. The velocity of the tortoise along AO = velocity along a = c Fig. 1.40 BO = velocity along CO = vcos 30 = ni : With this velocity, the three tortoise will meet at O. distance | AOorBO or CO Time taken by a tortoise = f= velocity = vV3/2 afi Se 2m V3 v¥33v On substituting the values of a and v we have 2.3 t=>x—=2s 31 aa You have either reached a page that is unavailable for viewing or reached your viewing limit for this book. aa You have either reached a page that is unavailable for viewing or reached your viewing limit for this book. aa You have either reached a page that is unavailable for viewing or reached your viewing limit for this book. 42 SOLVED PROBLEMS IN PHYSICS fm = reat "9 y o m |e 2 tl, = m 3 2g * Vv St Further [em = feat a Y o m{1_o1 3 fara] “8 ee MeL! ogy By? ic 21V? a = *2 "2 -. ..18 veo or ve 4 2.8. A ball of mass m starts moving from rest from the point O along the track OCB (see Fig. 2.6). Find the reaction 2 on the ball at C if the curve mx OCB is defined by equation y = h sin (=) where h = 7 _ (me Solution: y = h sin TT x 9 \ dy _ hn. (=) i ax TOI ! . i. dy 1 withx= > ieatG, 7° =O Ki ' 2 a? x wow to) : 8 mg y c ay hr x Fig. 2.6 atl, aaa Rap a@y aa You have either reached a page that is unavailable for viewing or reached your viewing limit for this book. aa You have either reached a page that is unavailable for viewing or reached your viewing limit for this book. aa You have either reached a page that is unavailable for viewing or reached your viewing limit for this book. 46 SOLVED PROBLEMS IN PHYSICS plateform as in Fig. 2.11. The system of the blocks is pulled in the forward direction by a force of 60 N applied on the block P. Find the acceleration of the system and tension in the rope at A, Band C. Solution; The common 4kg Sg acceleration of the system is cB given by F 60 = ———_ = — = 5 msec? Fig. 2.11 *= Totalmass ~ 12° 7 Tension at A is the force required to pull the block Q along with the rope. The total mass = 6 kg. T,=6x5=30N. Tension at B is the force requird to pull the block Q along with the portion BC at the rope. Total mass = 5 kg. T,=5x5=25N Tension at C= T,=4=x5=20N 2.14. What is the tension in a rod of length Z and mass M ata distance y from F, when the rod is acted on by two unequal forces F, and F,{< F): as shown in Fig. 2.12. (HIT 1978, 1993) Solution: The acceleration of the rod is given by a= (4-4) M Let Tbe the tension at O. For the motion of part OP we have ee pe Ok— y —+1 p x a —— _. —> a, By T=F, cya + 7 Fig. 2.12 2.15. Block A of mass m and block B of mass 2m are placed on a fixed triangular wedge by means of a massless in extensible string and a frictionless pulley as shown in Fig. 2.13. The wedge is inclined at 45° to the horizontal on both sides. The coefficient of friction between block 2 A and the wedge is 3 and that between block B and the wedge is 1 3 If the system of A and B is released from rest, find Fig. 2.13 aa You have either reached a page that is unavailable for viewing or reached your viewing limit for this book. aa You have either reached a page that is unavailable for viewing or reached your viewing limit for this book. aa You have either reached a page that is unavailable for viewing or reached your viewing limit for this book. 50 SOLVED PROBLEMS IN PHYSICS 2.20. A particle of mass m is projected vertically upward with a velocity v, in a medium whose resistance is kv. Determine (i) the time for the particle to come to rest (ii) the maximum height attained. Solution: (i) Equation of motion of the particle mdv —— =-(kv+m; dt ¢ 8) i mdu (ii) at" dh v=-(mg+ kv) . mudv_ __ mg+ky Put mg+ kv=X kdv= dX J mL) ay fan mg+kvy mg 2.21. Anauto vehicle ofmass m moves alonga horizontal track. Ata certain instant the engine is turned off. The resistive force that acts on it isa + pv where a and f are positive constants and vis instantaneous speed. Find (i) the time taken by the vehicle to come to rest (ii) the distance travelled by it. Solution: orh= ? [us — mg log, i) aa You have either reached a page that is unavailable for viewing or reached your viewing limit for this book. aa You have either reached a page that is unavailable for viewing or reached your viewing limit for this book. aa You have either reached a page that is unavailable for viewing or reached your viewing limit for this book. 54 SOLVED PROBLEMS IN PHYSICS On integration ~ 7 logimg+ rv) =t+¢, m Att=0, v=y,sina .- 7 log (mg + rv, sin a) ot ta Bagg [ma se) r mg +rv Att=t, v =0; “t= = log (+ For downward journey mdv at mdv mg —rv On integration we have rv, sina mg =mg-rv =dt -2 -log (mg-rv) =t+c Att=0,v=0; =~ & slog mg or t= tog me. } r mg -rv m, Att=t, v=v, sina; ® tog —78 __ r mg -rv, sina “Total time t= t, + 1.2 Jog | 2M sine «. Total time t= t, + t,= r log et me x Further s= > mg —ru, sina = mucosa |, _mg-ry,sina wiseiin mg tru, sina 2r mg +rv, sina ze ugsin2a TU, 2g|14+—2 si a +2 sina | aa You have either reached a page that is unavailable for viewing or reached your viewing limit for this book. aa You have either reached a page that is unavailable for viewing or reached your viewing limit for this book. aa You have either reached a page that is unavailable for viewing or reached your viewing limit for this book. 58 SOLVED PROBLEMS IN PHYSICS 2.28. The speed of a particle is determined in terms of distance s that it has travelled via equation v= v,— bs where bis constant. Find the value of Sand vas a function of time. ds Solution: v= ao bs ds oF ie = fat log (v, - bs) =- bt+C At t=0,s=0; -. C=log v, or log 0-2} =—-bt ( =e ye or s= Fp -e4) ds a7 Y=% e* 2.29. A jet of water with a sectional area A = 6 crm? strikes a wall at an angle of @ = 60° to the normal and rebounds elastically from the wall without change in velocity. Find the force acting on the wall if the velocity of water in the jet is v= 12 m/sec. Solution: Mass of water flowing per sec = Apu Momentum of water flowing persec = p=Apv* Incident normal momentum per sec = Api? - cos 60 Reflected normal momentum per sec = —Apv* cos 60 Force = 2 Apv* cos 60 = Apu? = 6 x 10+ 1000 x 144 tg 220 = 86.4N x 2.30. The potential energy ofa particle has the form U=a@ (z - | where ais constant. Find (i) the force acting on the particle, (ii) the work done on the particle is going from (1, 1, 1) to (2, 2, 4). Solution: @ oe te a and F, =~ =-% F=-—i+ te i-Ge aa You have either reached a page that is unavailable for viewing or reached your viewing limit for this book. aa You have either reached a page that is unavailable for viewing or reached your viewing limit for this book. aa You have either reached a page that is unavailable for viewing or reached your viewing limit for this book. 62 SOLVED PROBLEMS IN PHYSICS 2.37. Two spheres of masses m, and m, connected by a massless spring forms a system as shown in the Fig. 2.26. They fall under earth’s gravitational field with initial velocities v, and v, respectively. Calculate the total momentum and radius vector of the centre of inertia. Solution: The position of the centre of mass m m, is given by Yon = aa 3 y, and y, are the positions of the two masses. Fig. 2.26 (m, + m,) “ =Total momentum Ym, Me, & M m, at +m, a and are are the velocities of the two masses her time rf. =m, (v, + gt) + m, (v, + gt) = (mv, + m,v,) + (m, + m,) gt =p,+(m, + m,) gt; p, = total initial momentum. Dom _ MY + MV, dt m, +m, +e my, + mv, =U, + gf v= ° om +m, On integration we have 1 Ven = yt + 2 ge 2.38. A closed system consists of two particles of masses m, and m, moving at right angles to each other with velocities v, and v, respectively. Calculate (i) the momentum of the two particles (ii) the total kinetic energy in the frame of reference of centre of inertia. Solution: (i) Let us assume that particles of masses m, and m, are moving along x and y axis respectively. - TY x component of v,,, = ht — —Thv2 ycomponent of v,,, = msm, xcomponent of velocity of particle of mass m, in the reference frame my, ym, of centre of mass = v, - ———— = mtm m+m VU; Momentum of particle of mass m, along x axis = Tas m+tm, aa You have either reached a page that is unavailable for viewing or reached your viewing limit for this book. aa You have either reached a page that is unavailable for viewing or reached your viewing limit for this book. aa You have either reached a page that is unavailable for viewing or reached your viewing limit for this book. 66 SOLVED PROBLEMS IN PHYSICS d,=(R-1)-d, Now m [(R-~ 1) -d,] = Md, m(R-r) = Mem) (b) According to conservation of energy we have mg(R-1) = b mu; + $Me 2 1 MY», 1 or mg(R-1) = 3 m(-¥) Un + 3 Mv; M mg(R-1) = * (+) uy we 2m?g(R-1r) 7" M(M+m) [zea or =m \matem 2.42.. Two uniform thin rods A and B of lenth 0.6 m each and of mass 0.01 kg and 0.02 kg respectively are joined, end to end. The combination is pivoted at the higher end P as shown in Fig. 2.28a such that it can freely rotate about the point P in avertical plane. A small object of mass 0.05 kg, moving horizontally hits the lower end of the combination and sticks to it. What should be the velocity of the object so that the system could be raised to the horizontal position. (IT 1994) Solution: According to - conservation of angular momentum we have mv: 2l=Io v= velocity of the object where Jis the moment of inertia ofthe system about P. al I=1,+1,+1, I,=moment of inertia of bar B Mg B A about Pand is equal to =tMyP @Q-- Q-- 3 1 Fig. 2.28a Fig. 2.28 3 aa You have either reached a page that is unavailable for viewing or reached your viewing limit for this book. aa You have either reached a page that is unavailable for viewing or reached your viewing limit for this book. aa You have either reached a page that is unavailable for viewing or reached your viewing limit for this book. SOLVED PROBLEMS IN PHYSICS 70 Solution: (i) F=maz=F,cos wt dv fh cos @t a aes oO dt om du= A cos wt: dt m On integration, we have v= F.. sinot+A mo at t= 0, v=0;..A=0 hence v= am +sin ot mo Velocity v will be zero when t= = (ii) Time for the motion = = @ Now 2 = Fe dt mo +sin of ale fra = j= -sinot- dt é -F ne X= [8 -cos ot mo’ lo (iii) v= —> - sin wt mo dv dt bs This is possible with wt = 5 ort= on F, =0=— -cosat m * Upae = 4. ssin-o = “ma me 20 Ky mo 2.47. A motor boat of mass m moving on alake with velocity v, experiences a resistive force of water F = - rv. At an instant ¢ = 0 the engine fails. Determine (i) the time for which the boat will move after the failure ofthe engine (ii) the velocity of boat when it has covered a distance s. aa You have either reached a page that is unavailable for viewing or reached your viewing limit for this book. aa You have either reached a page that is unavailable for viewing or reached your viewing limit for this book. aa You have either reached a page that is unavailable for viewing or reached your viewing limit for this book. 74 SOLVED PROBLEMS IN PHYSICS Equation of motion is given by mdv dm “a ae 78 v ™ ie = J- v, a - fears m, is the mass of the rocket only. n v, = initial _— m vo vy=¥, log, 7 — Bt m or v=U,+ ¥, log, 8 (ii) Ascending rocket subjected to thrust only ie. F,,=0 Equation of motion is given by mdv __,, dm a" dt m, or fav =-v, an % mm v-v, =-v, log, m or v= y+, 0B, 2.51. A rocket of mass m, initially at rest moves by ejecting gas with a velocity u relative to the rocket. The external force is absent. Find its velocity when its mass becomes m. Solution: may 44 7 dt dt emt we dm 9 Tap 5 Featernat =O m d or |v = =-u f= mm = Mo v = ulog, = 2.52. A rocket of mass m, moves with a constant acceleration a in the absence of external force by ejecting gas at a velocity u relative to the rocket. In this process the mass of the rocket will continuously reduce. Find the mass of the rocket at an instant ¢. PARTICLE DYNAMICS 75 Solution: aye yp ma =~ UT + Fe adt _"-dm -—— = ;F=0 Pr > Law = at m or -— = log, — u My or m= met 2.53, A rocket of mass 3000 kg is set to fire in the vertical up direction. The gas is ejected at a speed of 500 m/sec. Calculate the mass of the gas ejected per sec in order (i) to overcome its weight (ii) to provide an initial acceleration of 9.8 m/sec’ in the upward direction. Solution: . dm (i) ma=-u > +mg=0 = Mi me “dt ou On putting the values, we have dm _ 3000x9.8 a = 58.8 kg/sec. . au. am Gi) m(g+a=u dt im ene) dt u On putting the values, we have dm _ 3000(9.8+9.8) _ _ ee =00 = 12 x9.8 = 117.6 kg/sec 2.54. A rocket of mass m, moves in the absence of external force by ejecting gas with exhaust velocity of 1.6 km/sec. For what ratio of the mass of the fuel to the mass of rocket it will attain a velocity of 11.2 km/sec. Solution: v =v, +0, log, m= 76 SOLVED PROBLEMS IN PHYSICS or =e m yr, mass offuel _ mMy—™ _ 9 _y = 1996-1 = 1095 ~ mass of rocket m 2.55. A cart containing m, mass of water moves under the action of a froce F along a horizontal plane. At the bottom of the cart there is a hole through which water comes out at a constant velocity of pp kg/sec. Assuming the initial velocity of the cart to be zero. Determine the velocity and the acceleration of the cart as a function of time by neglecting the frictional forces. Solution: Fd dv dm dv m dt dm dt dm © mat : V=F -lo o B inn v= = = Be My — pe a. acceleration = © x =! y nh a Bo my (my pt ne ™y — ut 2.56. A flat car of mass m, initially at rest moves by applying a constant force F on a horizontal plane surface (see Fig. 2.30). From a stationary hopper sand spills on the car at a constant rate of pkg/sec. Neglecting the force of friction determine the velocity and the acceleration of the car in this process of adding mass every second. Solution: da oer (mv) orFt=mv+A att=0,v=0 F -A=0 hence F-t= mv Now F-t=(m,+pd)u' Fig. 2.30 PARTICLE DYNAMICS 77 — Fp NE ut m,| 1+ ut) (+5 2.57. Asphere initially at rest rolls down a smooth hill consisting of curved and straight paths from the point O situated at height h above the ground (see Fig. 2.31). At what value of the height AB of the hill will sphere cover maximum horizontal distance x along the straight path BC. What is this distance equal to? Solution: Let AB = h,. In view of conservation of energy the velocity of the sphere at A is given by v= 2g (h-hh) wxevt= y2g(h-h) : =2/(h-h,)h, ; tis the time taken for the free fall distance h,. In the above equation x isa function of h,. Differentiate x with respect to h, and equate it to zero. orh-2h,=0 h orh,= 2 For this value of h, Fig. 2.31 x=2 2.58. A block of mass m = 1 kgis placed on A a plane inclined at an angle 6 = 60° with the horizontal pulled by a string attached 2 to it at an angle 4 with the plane (see & Fig. 2.32a). The coefficient of friction k between the block and the plane is equal to 0.14. Find the angle ¢ at which the tension T in the string will be minimum. What is it equal to? Fig. 2.32a 78 SOLVED PROBLEMS IN PHYSICS Solution: The various forces acting along and perpendicular to the plane are shown in Fig. 2.32b. The normal force N= mgcos 6-Tsin 6. Under equilibrium the forces along the plane are represented by the equation. mgsin 6 + f= Tcos ¢ where f, = frictional force =k (mgcos 6 -Tsin 4) On substituting the value of f, in the above equation, we have mg sin 6 + k-mgcos @-kTsin ¢=Tcoso _ mg sin0+kmgcos® ~ — (ksin > +cos 4) In the above equation Tis a function of >. Differentiate T with respect to $ and equate it to zero. dT _ —(mgsin®+kmg cos 6) (k cos > - sin >) (k sino + cos 6) Fig. 2.32b a a ‘ a will be zero only when k cos 6 -sin ¢=0 or tang=k=0.14 2 b=8° mg sin +kmg cos6 (Biji+e )+(1 i+) _ mgsin@+kmgcos® Vl+k? On substituting the values of m, g, @ and k, we have B 9.8x~3 +40.14x9.8x4 = —2__ Now T,,,. = PARTICLE DYNAMICS 79 2.59. A body in the form of a small cubical block of mass m can move freely without friction on a horizontal plane. The body is initially at rest and starts moving when a force F= at, « being a positive constant applied at an angle 6 = 60° with the horizontal at an instant t= 0 (see Fig. 2.33a). Find (i) the time at which the body will detach from the plane and (ii) the velocity acquired and the distance travelled by the body at the time of its breaking off the plane. F Solution: With reference to Fig. 2.33b the normal reaction force = mg - at sin 0. 9 When the body breaks off the plane the - normal reaction force is equal to zero. Thus mg=atsin®@ mg_ _ 2mg . asin@ a3 In other words the body will break off 2mg g a3 Fig. 2.33b From Newton's second law of motion, we have or t= ata time t= a a (mv) = a tcos 6 2 or mv=acos0 feat =a.cos8-— m’g? acos® mg*cos® orv=s——2,- . ———— = 2 a’sin’@ 2m 2a sin’ 6 _ mg? 2 _ mg? 2a %3 > 3a mis Further v= 4 = © cos 3 363 se cose ferae - acos6-t - acos® m 6m 6m a’sin’é _ m’g® cos ~ 6a? sin’@ mg 4 Imig © 6a* 3/3 9a*V3 2.60. A bar of mass m = 600 g lies on a smooth horizontal plane. The bar is initially at rest but starts moving along the plane when a force 80 SOLVED PROBLEMS IN PHYSICS F= 1.96 N (constant in magnitude) is applied to it. The direction of the force, making an angle B with the horizontal, is no longer constant but varies directly with x, the distance covered by the bar according to the equation f = ax where isa positive constant. Find the velocity of the bar. Solution: In Fig. 2.34, the driving force is Fcos B = 1.96 cos: (ax). According to Newton's second law of motion, dv F Force = m a 1.96 cos (ax) dv _ 1.96 B or at =m 8 (ax) = or HH NS cay dx dt m Fig. 2.34 or vdy 2 126 cos (ax) dx; oe =v m dt On integrating the above equation we have YP 196 in (any ory = | — 2x18 sing = | 22 sing 2 ma NO KY= Veo 10 xa * 15a 2.61. A body of mass m = 1 kg is slowly pulled up by a force F (constant in magnitude) along the surface of ahill as shown in Fig. 2.35. The coefficient of friction k between the body and the plane is 0.1. Find the amount of work done by this force if the height of the hill h = 4.5 m and the length of its base /=5 m. Solution: A force F is applied on the body of mass m. Under equilibrium we have F=mgsin 0+kmgcos 0 Let the body moves through the dx along the hill. The amount of work done by the force is given by F- dx = (mgsin @ + k mgcos @) dx dl ax= mg Zt sdx+ kmg a ax coso= and sin = Here dl and dh represent the displacement along x and y axis respectively. cous Fig. 2.35 PARTICLE DYNAMICS 81 a 1 Net work done = fF-ax = Jmgah+k mg fat =mgh+kmgl oO oO =mg(h+ kb On substituting the values of h, k, I, m and g, we have Work done = 1 x 9.8 (4.5+0.1 x5) =9.8x5 =49J 2.62. A body of mass M = 9.8kg with a small disc of mass m = 200 g placed on its horizontal surface ab, rests on a smooth horizontal plane (see Fig. 2.36). The disc can freely move along the smooth groove abc of the mass M. To what height (relative to its initial position) will the disc rise after separating from the body M when an initial velocity v = 5 m/s is given to it in the horizontal direction ? Solution: According to the law of conservation of momentum, we have mv = (M+ m) v’ (v' is the velocity with which the two bodies in contact are moving) ‘ mv eT in Fig. 2.36 Loss in the kinetic energy of the disc el i "2 = me 3 (M+m) v' ome rit =5™ -= (M+m) (M+m> es ae fi-—™) 1 mMv* "ig M+m) 2 M+m With this energy the disc will move up to a height h after separating from. the body M, i.e., nz % (Mt or ~ 2g (M+m On substituting the values, we have 5x5 98 2x98 * 19 7 15™ 82 SOLVED PROBLEMS IN PHYSICS 2.63. Body A is placed on a wedge forming an angle « = 30° with the horizontal (Fig. 2.37a). What is the acceleration that should be imported to the wedge in the horizontal direction for body A to fall freely ? Solution: Let an acceleration a be imparted to the wedge from right to the left. The distance travelled by the wedge in time ¢ = AB = (1/2) at?. During the same time ¢ the body falls D> freely through a distance AA'= (1/2) gf (see Fig. 2.37b). A ay Fig. 2.37a AB_a Ne oe 28 low cot a & “. @=gcota=9.8 x cot 30 =98x V3 = 16.97 m/s? Fig. 2.37b 2.64. A body starting from the top of a right-angled prism slides down along its hypotenuse AB (see Fig. 2.38a). The length of the base of that prism is |. The coefficient of friction between the body and the surface AB of the prism is k = 0.14. At what value of angle a will the body take the least time for sliding down the surface ? Solution: Various forces acting along and perpendicular to the surface AB are shown in Fig. 2.38b. Normal reaction force = N= mg cos a. Frictional force = f, = k mg cosa. Net downward force along the surface = mg sin a - k mg cosa. MB Net downward acceleration along the surface = g sin a - kg cos a. Time taken to cover the surface AB is given by Arg 1 Iseca= z (sina - kg cosa) t? 2l oke= Vgcin ‘a@cos a—Kk cos’ a) Here tis a function of a. Find out the Fig. 2.386 at value of da and equate it to zero. PARTICLE DYNAMICS 83 a = Jat a (sin « cos a - kcos? a)? g at 1/2 (cos 2a+Ksin 2a) 2 Ye (sinacosa—kcos’a)”* On equating it to zero we have cos 20 + ksin 2a = 0 1 1 or tan 2a=~ — 0.14 a= 49° 2.65. A right-angled triangular block Ahas a body Bof mass m placed on it asshown in Fig. 2.39a. Block A is given a horizontal acceleration a directed towards the left. What is the maximum value of this acceleration so that B is stationary relative to A? The coefficient of friction p is such thatp Mg the block B will leave the ground. Let H be the minimum height of o the block B' above the pan when the block B leaves the ground. Then at Fig. 2.47 that instant, Loss in potential energy of B'=Mg [=] Increase in potential energy of the pan = Mg- Now the block B will bounce off from the ground when Mg | -7 > Mg [#2] orH> a“ 2.74, A homogeneous and inextensible chain of length / and mass m lies on a smooth table. A small portion of chain of length x hangs from the table. Initially the part of the chain lying on the table is held and then released. Calculate the velocity v with which the chain will leave the table. Solution: With respect to the plane of the table, Potential energy = — E x-gxl2 As the chain slides we come across a situation in which the chain is exactly vertical and leaving the table with velocity v. Then Kinetic energy 1 isp > mv and Potential energy = ae vv ez 2Mg k PARTICLE DYNAMICS 91 In view of conservation of energy, we have }«—— (/-x) ——>| a | 7 *E EZ eo | or v= a( l Fig. 2.48 2.75. Two blocks of mass m and 2m connected through a massless rod slide on a plane inclined at an angle a. = 45° to the horizontal. The rod is parallel to the plane (see Fig. 2.49). The coefficients of friction k, and k, between the two masses and the plane, respectively, are 0.2 and 0.1. Find the common acceleration of the two masses. Solution: The equation of motion of the two masses along the plane is given by 2ma = 2mg (sina - k, cos a) - T; KN, = 2g k, cos a ma=T + mg (sin a—k, cos a); k,N,=mgk, cosa Here T stands for the tension in the rod and a for common acceleration. On adding the two equations, we have 3ma = 3mg sin a - mg (k, + 2k,) cos a “ @=gsina- 8 (k, + 2k,) cos a On substituting the values of the various terms, we have wo & x N a => = 6,02 m/s? 2.76. A block of mass m slides in a right-angled trough inclined at an angle a = 30° with the horizontal (see Fig. 2.50a). Calculate the acceleration of the block if the coefficient of friction k between the trough and the block is 0.141. Solution: A right-angled trough has two walls at an angle of 90° (see Fig. 2.50b). During motion there will be friction at the two walls. The total frictional force f, = 2 k.mg cos a cos 45 = 2 k mg cosa. 92 SOLVED PROBLEMS IN PHYSICS The equation of motion of the block is given by ma = mgsina-f, =mgsina— v2 kmgcosa A= (sin a- V2 kos a) On substituting the value of g, a and k, we have 1 2.x 0.141 a =98 [:- eis =8) “a —— 2 0.141x 173) yy =9.8 | 0.5 -——_—— 38 ( 1.41 = 9.8 [0.5- 0.173) Fig. 2.50a Fig. 2.50b .8 x 0.327 = 3.2 m/s? 2.77. Two blocks are tied through a string as shown in Fig. 2.51. Find (i) the force F exerted by the two blocks in equilibrium. (ii) the force F’ required to carry up the two masses with an acceleration of 2.2 m/s* and (iii) the tension in the string lying between the two blocks. Solution: (i) F=(2+4)x98=588N (ii) The equation of motion of the two blocks is given by “m F'-2g-T=2a T and T- 4g = 4a (ais the acceleration) On addition of the two we have 4kg =6(a+g) =6 (2.2+9.8)=6x 12=72N Gi) T= 4 (a+ g) =4 (2.2+9.8)=4x12=48N Fig. 2.54 2.78. A piece of ice slides down a plane inclined at an angle 6 = 30° with the horizontal in twice the time it takes to slide down a similar frictionless plane. Find the coefficient of friction k between the ice and the plane. Solution: Various forces are shown in Fig. 2.52. When friction is present the equation of motion of ice of mass m is given by mg sin 0 — kmg cos © = ma,; a, is the acceleration of ice along the plane or a, = g (sin 0 ~ k cos 8) Ifx be the length of the inclined plane, then 1 1 x59 % B= 2 g (sin 8 - k cos 8) t where t, is the total time taken by the ice to slide down the plane. In the absence of friction the equation of motion is given by ma, = mg sin 6; a, is the acceleration of ice along the plane. PARTICLE DYNAMICS 93 or a,=gsin@ The distance travelled in time ¢, is given by —_ x= 5 gsin@- tf v On dividing the two equations of x we have l= sin @—kcos@ (2): t, sin@ iA 1 _ sin6-kcos® 4° sind or 0.25 sin@ =sin@-kcos@ or kcos@=0.75 sin6 or k=0.75 tan® .75 tan 30 or Fig. 2.52 2.79. In the Fig. 2.53 the mass of the blocks A, B and C is 2 kg each. Neglecting the mass of the pulley and the string find the tension in the string connecting the two blocks B and C. Solution: The equation of motion of block Cis given by 2g - T = 2a; T is the tension in the string connecting the blocks B and C. Similarly the equation of motion of block B and A is given by 2g+T-T, = 2a; ais the common acceleration. and T, — 2g = 2a; T, is the tension in the string _T, connecting the blocks A and B. On adding the three equations, we have = of 2g=6aora= 3 se 2; Now T =2g-2a=2g- = _ 4x98 - 39.2 =13.07N Fig. 2.53 3 3 2.80. A massless thread passing over a frictionless and massless pulley contained two masses m, and m, at its end as shown in Fig. 2.54. Find the tension in the string if the pulley as a whole moves up with an acceleration g. Solution: Let a be the acceleration of the two masses 7m, and 17. Let us assume that m, is going down and m, is going up. Now m, will go 94 SOLVED PROBLEMS IN PHYSICS downwards with a resultant acceleration a - g and m, will go up with a resultant acceleration a + g. The equation of motion of the two masses is given by 9 m, (a— g) = m, g—T; Tis the tension in the string and m, (a+ g) = T- mg On adding the two equations, we have (m, + m,) a=2(m,-m,) g§ _ 2(m, -m,)g ora= m,+m, tT Now T =2mg-ma (m,-m,) « mf =2m,g-2m, m +m, | or T= 4mm & mH ~"'m, +m, Fig. 2.54 2.81. In the arrangment shown in Fig. 2.55a the plane on which the block Amoves is inclined at an angle 6 = 45° with the horizontal. The coefficient of friction k between the block A and the plane is 0.141. The ratio of the masses of the blocks Band A “2 == 3 Initially the systemis stationary. Find the acceleration of the Bidck Bwhen the masses start moving. Solution: m, = n m, (given) The various forces acting on the mass m, are shown in Fig. 2.55b. Let abe the common acceleration of the system. The equation of motion of the two blocks is given by T- m,g sin 6 - k m, g cos 8 = m, a; Tis the tension in string and nm,g-T=nm,a On adding the two equations, we have m,g(n-sin @ - kcos @) = (1 +1) m,a Fig. 2.554 PARTICLE DYNAMICS 95 ora= Bi (ny - sin 0 - k cos 6) n+l On substituting the values of 6, k, 1 and g we have 98 {3 1 1 = |=-—-0.141x— ora [3 v2 ZI 2 ae [1.5-0.707-0.1] = s2n0609 a 0.693 _ 2.71 mis? 2.82. In the arrangement shown in Fig. 2.56, block A moves on a frictionless plane inclined at an angle a = 30° with the horizontal. Neglecting the masses of the pulley and the connecting strings find the acceleration of block B whose mass is n = 2 times the mass of block A. Solution: Let m and nm be the masses of the two blocks A and B respectively. The equation of motion of two blocks is given by mg sin a—T=ma,; a, is the acceleration of block A. nmg - T, = na; ais the acceleration of block B. -a On substituting a, = z in the first equation, we have -mgsina + T= or ma=2T-2mgsina and y ma= S +7 mg since T=2T, On solving the two equations, we have 2g (2n - sin a) 4ytl On substituting the values of a and n, we have 2gx 3. 5 9 g = 7.62 m/s* Fig. 2.56 2.83. Calculate the acceleration of the system and the tensionin the strings for the system shown in Fig. 2.57. Solution: The equation of motion of mass 8 kg is given by 8g-T, =8a @ Similarly equation of motion of mass 2 kg and 4 kg is given by a= a= eis 96 SOLVED PROBLEMS IN PHYSICS T,- T,- 2g cos 60 = 2a did) and T, - 4g cos 60 = 4a (iii) On adding the three equations we have 8g- 6gcos 60 = 14a orl4a =5g = 58 “14 _ 5x98 2 nil “Yq Fees Using equation (i), we have T, = 8 (9.8- 3.5) =8 x 6.3 =50.4N Using equation (iii) we have T, =4a+ 4gcos 60 = 4a + 2g =4x35+2x9.8=33.6N 2.84. In the system shown in Fig. 2.58 block of mass m, slides on a frictionless plane inclined at an angle 6 with the horizontal. Neglecting the mass of the pulley and connecting strings, find the acceleration of block of mass m, moving up along the plane. Solution: In the arrangement shown in the Fig. 2.58 mass m, has a displacement twice that of m,. The acceleration of m, will therefore be twice that of m,. Let a be the acceleration of m,. Then the acceleration of m, is a/2. The equation of motion of mass m, is given by ara me i mg-2T= @ Similarly the equation of motion of mass m, is given by T-mgsinO=ma (ii) One eliminating T from these two equations 2g (m, - 2m, sin 0) se S Fig. 2.58 4m, +m, 2.85. In the system shown in Fig. 2.59 blocks B and C of mass m, and m, rest on a horizontal plane. The coefficient of friction between the blocks and the plane is k. Neglecting the mass of the pulley and the connecting strings find (i) the acceleration with which block A of mass m,, comes down and (ii) the tension T in the string connecting the two blocks B andC. PARTICLE DYNAMICS 97 Solution: Let a be the acceleration of the system. Equation of motion of block Cof mass m, is given by T-m,gk=ma (i) Equation of motion of block B of mass m, is given by T,-T-m, gk=ma (ii) Equation of motion of block A of mass m, is given by mg-T,=ma@ (iii) On adding the three equations, we have gim,-k(m, + m,)] =(m,+m,+m,)a _ 8 [mp —k(m, +m) My +m, +m, On substituting the value of a in equation (i) we have T =m,a+ mgk= m, {a+ gk) m je wae +m,)+gkmy + gkm, + gkm, My +m, +m, or a Fig. 2.59 _ gm,m,(1+ k) mM, +m, +m, 2.66. A frictionless cart of mass M carries two other frictionless carts connected bya string passing over a pulley as shown in Fig. 2.60. Calculate the force which, when applied to the cart of mass M, will prevent the two carts of mass m, and m, to move relative to it. Solution: Let a be the acceleration when the two carts of mass m, and m, are moving. Now we have ma=T=m,gora= mm g Rrequired force F = (m,+m,+M)a My =(m,+m,+M) 7 8 Fig. 2.60 2.87. In the arrangment shown in Fig. 2.61, the bodies have masses m,, m, and m,, friction is absent, and the mass of the pulleys and the threads is negligible. Find the acceleration of body m,. Solution: Equation of motion of mass m, is given by m,a,=T (i) Equation of motion of masses m, and m, is given by 98 SOLVED PROBLEMS IN PHYSICS mg-T,=m, a, Gi a-& T= 2T, and 7 34 T,-m,g=m,a, (iii) On substituting T, = T/2 = m, a,/2 in equation (iii), we have m,4a,-2m,g=2m,a, (iv) Similarly on substituting 7, = m, a,/2 in equation (ii), we have 2m,g-m,a, = 2,4, (vy) On substituting a, as (a, - @,)/2 in equations (iv) and (v) we have m, (a, -4,)-4m, g=4m,a, and 4 m,g- m, (a,-4,) = 4m, a, On solving these two equations, we get ane [ie + hy (Mm | oN Lamm, + my (mm, +m,) 2.88. In the system shown in Fig. 2.62 the mass ratio of the ball and the rod (m,/m,) = 1 = 1.5. The length of the rod /= 1 m. The ball is released after setting it at the level of the lower end of the rod. Find: (i) the acceleration of the ball, and (ii) the time after which the ball will be at the other end of the rod. Solution: In view of the arrangement shown the mass m, is displaced twice as fast as m,. Therefore its acceleration will be twice that of m,. Let a be the acceleration of m,. Then the acceleration of m, will be 2a. The equation of motion of the ball and the rod is given by Fig. 2.64 2T-mg=ma and mg-T=2m,a On solving the two equations, we get eT a | a - iT. «-(ieii)s “ue m, 2-n me = (2 g& Fig. 2.62 = 28 x 9.8 = 0.89 m/s? " . 1 In view of the equation of motion s = ut + 7% e aa You have either reached a page that is unavailable for viewing or reached your viewing limit for this book. aa You have either reached a page that is unavailable for viewing or reached your viewing limit for this book. aa You have either reached a page that is unavailable for viewing or reached your viewing limit for this book. aa You have either reached a page that is unavailable for viewing or reached your viewing limit for this book. PARTICLE DYNAMICS 103 KL cos @= mg; L= RG +07 — 1] (see Fig. 2.67b) or S08. [(F+x -1} E =mg Px? or 5 [Ve +x -1] = JP4x or 4.J/P +x? =51 or 16]? + 16x? =257? or gal! “4 According to conservation of energy. Gain in K-E of A + gain in K.E of B + gain in PE of spring = Loss in RE of B dnt mite tits gmt 5 mut + © KL = mgx or mot > 276 [eax -1f =mg. > 2 5mg 2, ol 3mgl 24, —— — o—_ or mv?+ 1 [fe Lh =: 4 5mg _T? _ 3mgl 2 — = or mu*+ 21 x 16 4 3gl Sgt PS ee or eg 32 _ 19h ~ 32 _ isa ve V2 3 Circutar MorTION AND GRAVITATION 3.1. A particle moves around a circle of radius 15 cm with a constant tangential acceleration of A, = 2.5 cm/sec’, Calculate the time from the beginning of the motion at which the normal acceleration A, will be equal to the tangential acceleration of the particle. Solution: v A= . ne Fr rs A, 0.15 ts AE = 25x10" = 2.45 sec. 3.2. A small particle moves around a circular track of radius 15 cm with a constant tangential acceleration A,. Calculate A, if the particle acquires a velocity v = 88 cm/sec after completing four complete revolutions. Solution: de 7 Aire radius of the path wont ae fao =. fe-ar r e -44 +C Att=0,6 =0 c=0 At CIRCULAR MOTION AND GRAVITATION 105 v On putting t=— A, vy ~ Or-A, P «> Qnn-2r 88x10“ ~ 2x3,14x4x 2x 015 = 0.103 m/sec.? 3.3. A small particle moves around a circular path of radius r= 15 cm with constant tangential acceleration A,, Calculate the normal acceleration of the particle in time ¢ = 15 sec in which the particle has completed 5 revolutions and has acquired a linear velocity of 20 cm/sec. Solution: In the light of question No. 3.2: 2nn A - @= Be -tand6= Bk r ar A = Apt -(22) e “or Ol te r Further v= re =A,-t (ut)! = (4nnr? (4nnr)° A ve = 16n'n'r? (0.2)* x 225 ~ 163.14? x 25x 0.15° = 0.027 m/sec.? 3.4, A small ball of mass m is attached to one end of a massless elastic string of unstretched length /, and elastic constant K. The ball is rotated in a horizontal circle with an angular velocity , by holding the other end of the string. What amount of work will be done in increasing its angular velocity to w,? Solution: K(l-1,) = mlo? 1[K-mo?] = Ki, pe Kh * K-mo? 106 SOLVED PROBLEMS IN PHYSICS 1 = Initial energy = 2m (lo,)? + 5KU- 1)? 1 1 268 =3 mP ow a K: a =; 7 aS iN [K+ mor] a1 moj RB (K+moi) 20K (K- ma?) =! meee Mme) =3 mKa & (K-mai) Further K(1,-1,) =ml,03 1 E,= Final energy = 2 m(1,@,)? + + Ka L~ by)? = 1 mK ot (+m) = 3 mKh o: (K-mo?) (K+moz)_ , K+mo? 1 2 le? =B- == oO, ——_— Work done E,-E, 2 mK; | ® K-me? 1 (F_met y 3 3.5. A particle travelled 7 th ofa circle ata mean speed (v) during a certain time. Find the magnitude of the mean velocity (V) of the particle during the same time. Solution: Suppose the particle is moving on the circumference ofa circle of radius R and completing one revolution in time 1. 3 2nR 2nR Mean speed (Qe =. ar 2 4 Magnitude of mean velocity (V) in that time = ae = ae ze Tt 4 iv? w) , W2 yy 3 2x 3m 3.6. A small body of mass m slides without friction from the top of a hemisphere of radisu R as shown in the Fig. 3.1. At what height will the body be detached from the surface of the hemisphere ? Solution: Let us assume that the body is detached from the hemisphere after traversing a vertical distance h. At B the velocity is given by CIRCULAR MOTION AND GRAVITATION 107 Vv =u’ + 2gh since u=0 v= /2gh At the point Ba component of the weight, i.e., mg cos 6 will be equal to the centripetal force since the reaction force at B will be equal to zero. mv? Aga or mgcos@ = —— or 8 RT RR or R-h=2h R = 0. or h=g Fig. 3.1 3.7. Asmall sphere of mass m attached toa string of length /is suspended from O (see Fig. 3.2). A peg P is located at a distance x below the point of suspension O. What should be the value of x if the ball starting from A swings full circle with the peg at its centre. Solution: In order to swing , completely in a circle centred on the peg P the sphere must have a speed J/g (I- x) at the point C. In . view of conservation of energy the \ # total energy at C is given by the nN ¢ potential energy mgl. Therefore, \ ' | x 10” mal = ; mg (I-x) +2 mg (l-x) "Ss or 2l=(l-x)+4(l-x» Sees or 5x =3! @ baose a eeod 2 3 = =1=061 or x 5 3.8. A small sphere of mass m attached to one end of a rubber cord of length / revolves in a horizontal circle centred at the other end of the cord with a frequency n. The tension in the cord is no longer constant but varies directly with elongation. Find (i) the radius of the circular path (ii) the tension T’ in the cord by taking tension per unit elongation as k. Solution: The sphere revolves in a horizontal circle. The tension in the cord is balanced by the centripetal force according to the following equation: Tension f= mre? = mr- 4n’n* where r is the radius of the circular path. dt =4r? mn? dr 108 SOLVED PROBLEMS IN PHYSICS Let Tbe the tension in unstretched condition, i.e., T = 4x? mn? | T+dt Tension _ dr ~ elongation — EF, @ % dr dr or T=(k-4n’mn?) dr T 2. d= — ”° atin or kT _ k-4n*mn'l k—4n'mn? ~ k-4n*mn? Further T'= T + dt = 4n°?mn?l + 4x?mnidr = 4x’mn?R where R is the radius of the circular path = 1+ dr a ) 7° 4r?mn? ~ k-4n?mr? 3.9. Asmall sphere of mass m slides down a plane from point A at height .H and then moves in a circular loop of radius r as shown in Fig. 3.3. Calculate (i) height Hin terms of r, and (ii) pressure exerted by the sphere at P. Solution: If the ball has to Ay cross C it must have a critical speed of Jgr . Thus the total H 5 energy, mgH = mer Fig. 3.3 we ot or H= > «T's T+ dt=k-dr= energy at Cis given by 1 mgr + mg: 2r In view of conservation of The kinetic energy at P can be obtained as 1 sr 3” Vi +mg(r+rcos 0) =mg- V, is the velocity of sphere at P. or V; =5gr—2gr—2grcos® or V% =(3-2cos6) gr At P we have mV; mgcos0 + F= = (3-2cos 0) mg; Fis the force of reaction at the point P or F=3 mg (1-cos 8) CIRCULAR MOTION AND GRAVITATION 109 3.10. A sphere starting from point A moves in a vertical loop from which a symmetrical section of an angle POR = 20 has been taken out (see Fig. 3.4). From what maximum and minimum heights should the sphere be allowed to move so that after losing contact with the track at point P, it reaches point R via a parabolic path PQR in air? Solution: Let v be the velocity of A the sphere at point P. In view of the figure, PR will be the range of the parabolic path PQR. ~. Range = PR = 2rsin 8 The time of flight for the 2usin8 parabolic path, PQR = suse 2usin® Fig. 3.4 Therefore, 2r sing = —“™" x ycos 0 y= ST ae cos@ According to law of conservation of energy we have 1 mgh = mgr + mgr cos @ + 2 mv Substituting the value of v” from the above we have mgr mgh = mgr (1+ cos 8) + Decse h 1 — cos 6 = cos + cos’? 6+ > Tr 2 h Put 7 =b Now 2 cos* 0-2 (b—1)cos@+1=0 coso = 2 = DEVE — 1-8 25 [-p+ (o-F -2] 4 for @ to be real, we must have (b- 1) > V2 or b21+ V2 For cos @, we have the identify 0 < cos @ < 1, i.e. b-1+ Ve-0? -2 <2andb<2.5 ie, 1+ J2 cos8+ > mv Fig. 35 H H H or mg|— | =mg | | cose +mg |Z | cos@ or 5 = cose 2 or cos 8 = The velocity at the highest point of the trajectory after breaking off the groove equals 1 vcos® = Jy cos?@ = [8H cos’ @ = fpgy(2) = [ag 2 gH = \ae(3) 27 38 (V3 3.12. A track shown in Fig. 3.6 consists of a straight portion of AB and a semi-circle BCD of radius r. A particle of mass m moves to the right along 22, the straight path with a constant velocity (FE. After covering the straight track it moves up and after some time it reaches point O where it loses the contact with the track. How far from B will the particle strikes the straight track AB? 2 Vo r Solution: At O we have =mgcos 0 CIRCULAR MOTION AND GRAVITATION lll or Y= Jercosé According to conservation of energy we have 1 22gr 25 On solving cos 6 = 4/5 and sin 6 = 3/5 x=rcos@=0.8r =5 mer cos 0 + mgr (1 + cos 6) For height EF we have (Vgrcos6-sino)’ = 2gh orh =EF= 2222 gine g _ 18 = 125 _ 18r = 243r Fig. 3.6 = 995 BETH 55 \ 6-sin® Time for path OF = aaa =t Distance OE= Jer cos 6 -cos @-,/gr cos6-sin 8 =rcos’ @-sin® Distance EL = r sin 6 - rcos? 6 sin 6 = rsin? 6 = te Time of flight for path FB 243r 1 5 ps 28% 2x 243r 2h = 125g 2 x 24. Distance EG = Jer cos §-cos@- — 4 [2x243 * 5 WV 125 5 T fexoas ar (2x243_ 27r 125. ~—«125 = 1.19497 aal8 e ~ nN a Distance BG = aa You have either reached a page that is unavailable for viewing or reached your viewing limit for this book. CIRCULAR MOTION AND GRAVITATION 113 At Psum of the forces fsin 45 + Nsin 45 will be balanced by the weight of the particle, i.e., fsin 45 + Nsin 45 = mg or (wN+N)=mg V2 or N(1+p)=mg V2 For the purpose of rotation the necessary centripetal force is obtained from the force N cos 45 — fcos 45, i.e., Nos 45-fcos 45 = mro? or N (L- p) = V2 -mw? R cos 45 (r = R cos 45 where R is the radius of the bowl) = ma?R On dividing the two equations, we have oR “(2 “) gVv2 \lt+n ei la-w gv2 (+wR On substituting the values of the various terms, we have o= 0.5 x 10v2 A sar 15x10v2 — V3 3.15. A particle of mass 100 g is suspended from one of the end of a weightless string of length 1 m and is allowed to swing in the vertical plane. The speed of the mass is 2 m/s when the string is inclined at angle 6 = 60° with the vertical. Determine (i) the tension in the string at 6 = 60°, and (ii) the speed of the particle when it is in the lowest position. Solution: At the point B (Fig. 3.9) the equation of motion of the particle is given by Us T — mg cos 60 = (ris the radius of circular path = length of the string, T is the tension in the string, v, is the speed of the particle) 2 p= ee +mgcos 60 =i xa L1E28 = 0.40 + 0.49 = 0.89 N 114 SOLVED PROBLEMS IN PHYSICS The total energy at Ais Kinetic and is given by 2 mu’, = mg (r- rcos®) + 4 > 3g MURsY, is the speed of the particle at A 2 = aie % ays faxaextea = Jigs =3.715 m/s 3.16. A light metallic chain of mass m = 50 gm and length / = 60 cm is fitted on an ebonite disc after its ends have been soldered (see Fig. 3.10a). Find the tension in the chain when the disc rotates with a speed of n= 60 rps about an axis passing through its centre. Solution: Mass per unit length of the chain = — Consider an element of the chain of length R - 2 6; R = radius of the disc Mass of the length sought = 4 R20 Let T be the tension in the chain. On resolving T in two mutually perpendicular directions, the component T cos 0 will vanish (see Fig. 3.10b). Thus we have 7 R200?R= 2T sin 6 = 2T 0 (sin @ = © when 0 is small) O- Fig. 3.10a Fig. 3.10b m m or T= R 4r? n? = T nP; 4? =P =mli-n? On substituting the values of the terms m, | and n, we have T= 50 x 10° x 60 x 107 x 60 x 60 = 108 N 3.17. A sphere of mass m is suspended from a string of length / from the point O (see Fig. 3.11). The sphere rotates in a circular path in a horizontal CIRCULAR MOTION AND GRAVITATION 115 plane. The string makes an angle a. with the vertical. Find the time period of rotation. ° Solution: For the rotation of the sphere in the l\ circular path a centripetal force m Ro? is required. Further, mRo? hy r = tana mg or o = SZ tana= 8x2 28 G---LAS R m 2 Rh h - 2. fe rT=2 f Fig. 3.11 or O = A y_ THe le ig. 3. 3.18. An object is placed at the top of a hemisphere of radius r. What horizontal velocity should be imparted to it so that it may detach from the surface of the hemisphere at the initial point of its motion? Solution: Let v, be the velocity imparted to the object. Let its velocity be v, after it has decend through h. According to conservation of energy we have cA S 3 m, uy + mgh= ; mu; (see Fig. 3.12) yy or vi =v? +2gh D At the break off we have Fig. 3.12 mv; — = 8 7 = mgcos “ v =groosO=gr [=| =v? +2gh or gr-3gh= uv; At the initial point h=0 2s Ne 3.19. A particle of mass m is projected from the point A with a velocity of Y= : 95gr along the inner surface of a smooth vertical circle of radius ras shown in Fig. 3.13. At what angle @ from the vertical it will detach from the track and what will be its velocity at that instant? Solution: (i) According to conservation of energy, we have 1 ; moe -95 gr=mg [r+ ros + me 116 SOLVED PROBLEMS IN PHYSICS 2 Further mg cos 6 = “ 19 1 10 =1+cos0+ 3 cos 6 93 = 5 cos® 1 3 0=cos! = co: 5 Fig. 3.13 (ii) v?= grcos = = gr aie J ves 15, 5 gr 3.20. An insect is sitting on a gramophone record rotating on its base at 100 as revolution per min. The insect is at a distance of 9.8 cm. from the centre of the record. Calculate the coefficient of friction between the record and the insect. Solution: pg = mro* _ ro? . 4n-n’r & & On substituting the valves we have 4nv | 25 u = foe) ai x 9.8 x 107 ne 81 3.21. A ball of mass m = 250 gm. is fixed at one end of a 1 mlong string. By holding the other end of the string the ball is rotated in a horizontal circle. Assuming the greatest weight that the string can support is M=4kg. Calculate the maximum number of revolutions/sec that the ball will make without breaking the string. Solution: T = Mg= mlw* Ye 2nn= (4) ml swwdes (8° ml CIRCULAR MOTION AND GRAVITATION 17 On substituting the values we have 1 ( 4x9.8 y * 2x3.14 (0.25x1 = 1.99 = 2 rev/sec. 3.22. Two identical balls of mass m are tied to two weightless strings of equal length / as shown in the Fig. 3.14. The system thus formed is rotated in a horizontal circle with A as centre at a constant angular speed w. Find the ratio of the tension in the two strings. Solution: The equation of motion of ball Cis given by T, = m-2l- 0? The equation of motion of ball Bis given by T,-T, = ml-o? or T, = T,+mlo? =m-2l-o*+m-1-0? =3ml-o? T _ 3mlo* _ 3, Fig. 3.14 T, 2mlo° 3.23. A car is moving on a circular road of radius 500 m at a speed of 25 m/sec. It is increasing in speed at a rate of 3 m/sec?. Calculate its acceleration. Solution: 2 25x25 Radial acceleration a, = Ye OS 25 m/sec.” r 500 2 3 dv Tangential acceleration a, = a 3 m/sec? :. Resultantaccelerationa = fa? +a? = 1.5625+9 =3.25 m/sec? 3.24. At what speed earth should rotate about its own axis so that a person 2 on the equator should have his weight as 5 of its present value? Solution: 2 5 mg = mg- mro?; r= radius of the earth 38 = ro aa You have either reached a page that is unavailable for viewing or reached your viewing limit for this book. aa You have either reached a page that is unavailable for viewing or reached your viewing limit for this book. 120 SOLVED PROBLEMS IN PHYSICS The velocity at rebound = v; = 30, el ogg = 3 12 al Now in view of conservation of energy we have mg {I-Icos 6] = ; m- ; 2gl 1 = 4 cos§ = 4 cos @ = 0.75 @ = 41°25’ 3.29. In the simple pendulum shown in the Fig. 3.18 the mass of the bob m= 170gm and the length of the string /= 1 m. The bob initially at Ais at rest. When the bob is released it swing through an angle 6 = 30° and strikes a spring of force constant K = 1960 N/m at B. Find the compression in the spring. Solution: According to conservation of energy mg [l-1cos 30] = ; Kx; xis the compression in the spring no(-S]-3 mgl [2-v3] = Ke zs [mgt — On substituting the values of various terms we have x= =1.5cm Fig. 3.18 3.30. A particle travels along a parabolic path given by x* = 4ay with a constant speed v. Calculate the value of the maximum acceleration acting on the particle. Solution: # Y= aa ay Xx a” 2a @y 1 E ¥ CIRCULAR MOTION AND GRAVITATION 121 The radius of curvature p is given by ay a = ax? = 2a 2 21% aly [ee] For acceleration to be maximum p should be minimum. The same will be obtained by putting x= 0. ade p or acceleration Dl v? 2a 3.31. A particle is projected with an initial velocity v at an angle 6 with the horizontal in the earth’s gravitational field. Calculate the radius of curvature of the path when the particle is at the highest point of its trajectory. Solution: The particle will describe a parabolic path given by 2 reIS RH axtune- ie — = —_ = 2 § Feos'6 @y we dx sang v* cos’ @ fy ___8 dx? v? cos’ 0 dy 1. oa - 2 % (2) ax dy j ‘cp . . The value of ax 2 the highest point is obtained by putting x = Range as 2 vu + =— -sin®@-cos@ & * — sin@-cos® Now = tanO-g- - —— = 122 ae 1 v? cos’ 0 p= = ay g ax? SOLVED PROBLEMS IN PHYSICS 3.32. A particle moves in xy plane with a velocity v whose magnitude is constant and describes an ellipse with semi major and semi minor axes as a & brespectively. Find the acceleration of the particle and the radius of curvature of the ellipse at the point x = 0. Solution: The equation of ellipse is given by ete =1 ei yest ya'-x v' we LD _ yh, & ag (FE) oe ae, mae dt” dt vasa Radius of curvature r= foelaation = BF 3.33. A particle initially at the origin moves in xy plane with a velocity V=ai+ bxj. Where aand bare constants. Find (i) the path of the particle (ii) the radius of curvature of the trajectory. Solution: @ V = air bxj & = velocity along x axis =a dx = adt or x=at 2 = bx lt dy =b-at-dt aa You have either reached a page that is unavailable for viewing or reached your viewing limit for this book. 124 SOLVED PROBLEMS IN PHYSICS The scooter will ride without sliding when Kmg cos 0 = me or P = Kpgcos0; & =tand ax v x 2 rel <- cos? Bl B B 1 or 2 = SS. 5s © sin{ x)P B B [rs Fe-coe (a) Be or vs “ee or vs p- [Xe 3.35. A ball A of mass m moves along a parabolic path y= s as shown in Fig. 3.20. Calculate the reaction force of the track on the ball when it is at the point O. Solution: As the ball decends through a its velocity v is given by v= /2ga y axis dy _8ax t ae a RI ay R = ey 8a oa OE 3 a) 2 (t= 2 Fig. 3.20 [+ ax 1+ b Atthe point O x=0 1 8a Dl ae At the point O we have R-mg = mat ami-rgaw 8 Pp b 2 R = mg [1-18] CIRCULAR MOTION AND GRAVITATION 125 3.36. A 3 kg ball is attached at the middle of a 2.828 m long weightless string as shown in Fig. 3.21. The ends of the string are attached to the two points ofa rod A and Bwhich are 2 m apart. The ball is rotated in a vertical circle at a speed of 4 m/sec. Find the tension in the string when the ball is (i) at the lowest point (ii) in the horizontal position (iii) at the highest point. Solution: (i) In the lowest position, we have 2 2r, cos 8= mg= mg ee eS r 2cos@ On substituting the values of various terms we have WW : 2 3x9,8+ 3%4 NAA = Sura Fig. 3.21 =27.37N. (ii) In the horizontal position we have 2 2T, cos @ = = 2 _ _mv "2" 2reos0 On substituting the values we have ___ 3x8 2” 2x1x1414 =16.97N (iii) At the highest point we have 2 2T, cos 6 = —_ -mg mv =mg ay F 3 2cos0 On substituting the values we have 3x4? -3x9.8 T= Syria =6.57N 126 SOLVED PROBLEMS IN PHYSICS 3.37. A thin circular wire of radius r containing a bead of mass m rotates about a vertical axis with an angular velocity . Find (i) the angle 6 at which the bead rests relative to the wire (ii) the normal reaction force on the bead. ' Solution: Ci yi t (i) Let Nbe the normal force then in view of Fig. 3.22 we have Nsin0 = mo*rsin® «N= mro? morsin® or tan® = ————— = — «sin0 mg g& = & cos 8 = or C 8 = cos! & oT gs |} rm (ii) Nsin@ = mor [:- 2 7 Fig. 3.22 or 3.38. Two balls of masses m and 2 m suspended through two strings of same length | forms two pendulums as shown in the Fig. 3.23. The pendulum of mass mis kept at rest at an angle of ®, = 45° with the vertical. When the ball is released from its rest position it strikes the ball of the second pendulum elastically. Calculate the angle 6, through which the second pendulum will be deflected. Solution: Let v, and v’, and v, and v’, be the velocities of the two balls before and after impact then we have mu,+2m-0 = mv’, +2mv', or v'+2v', =, (1) According to conservation of energy we have 1 1 J, 1 = mu +> +2m-0= > mul? += - 2m? 2 2 2 or vp = vu? +22 ... (2) Now uj -u)? =2 v? and yu =2v, On dividing the two we have yt, + (3) vtu-2v, = or 2u, or v= ww (4) CIRCULAR MOTION AND GRAVITATION 127 In view of conservation of energy we have ; mv; = mgl (1 -cos 45] = mgl [»- + or y= yet[2-v2 | Again in view of conservation of energy we have ; +2m- vi? =2m-gl{l-cos0,] or sae = gl{1-cos6,] or 2 xgt [2-v2 | = gl[1—cos 0,] 2 x 0.586 = 1-cos@, 0.13 = 1-cos®, cos @, = 0.87 0, = 29°30’ 3.39, A smooth hollow tube PQ as shown in Fig. 3.24a is one fourth arc of R acircle of mean radius R. The tube is filled with a chain of length > and mass m. The chain is released from rest. Find the velocity with which it will move along the frictionless horizontal plane QG after emerging out of the tube. ° D 7D oO <4 Fig. 3.24a Fig. 3.24b Solution: In Fig. 3.24b, consider an element of length dL, such that dL? = dx’ + ay Fora circle x* + ¥ = R° 1. xdx = -ydy a dy=-~-dx very 128 SOLVED PROBLEMS IN PHYSICS Now dL. = fact + de oF ay y y y component of centroid is given by R R Y= fh ya = fy = 2R ~ fia “RR ° = 2R Thus the y component of centroid is at a distance of from the centre of the arc. 2R Nowin Fig. 3.24a, the centroid is at a height of (r - 28) above Q.In view of conservation of energy we have 1 2) = = »R[1-= 2 mv? = mg r( x = {0.727gR = 0.85 /gk 3.40. A smooth hollow semi-circular tube of mean radius R completely filled with a chain of mass m is placed in a vertical plane as shown in the Figure 3.25. A slight disturbance causes the chain to move. Find the velocity with which the chain will emerge from end Q of the tube. 2R Solution: The y component of centroid is at a height =“ above O (see Question No. 3.39). According to conservation of energy we have dite mg. 2 amg. 2 2 =mg T e ve e[tenle nt = {4414 gR xa v=2.1 /gr Fig. 3.25 3.41. A conical pendulum of length /= 40 cm rotates at constant speed v in a horizontal circle of radius r = 20 cm as shown in the Fig. 3.26. How much of the string must be pulled through the tube to double the speed of ball? CIRCULAR MOTION AND GRAVITATION 129 Solution: In view of the forces acting on the ball we have 2 Tcosa = mgand Tsin a = = tana = e aot * B yp? Similarly tan a'= rE ; v’is the velocity of the ball when it moves ina circular path of radius r’. According to conservation of angular momentum we have 1,0, = 1,0, v uv or mr?- = mr? -— r =r’. 2v (v'= 2v given) me=rev ver Fig. 3.26 On putting the value of r= 20 cm pz 20N67 _ 5V67 ~ 6. 4 Length of the string pulled = /- /'= 40 - 10.2 = 29.8 cm. 3.42. A ball of mass m is resting on the straight portion of the track shown in the Fig. 3.27 at a height h about the lowest point A.When the ball is released it enters into the circular part of the track of radius r after covering the linear portion of the track. Find (i) the kinetic energy of the ball when it is at the point O' (ii) the radial and the tangential accelerations when the ball is at O’. Fig. 3.27 =10.2cm 130 SOLVED PROBLEMS IN PHYSICS Solution: (i) The kinetic energy of the ball at O' is equal to the change in potential energy. .. KE. of the ball at O’= mgh— mg [r+ rsin 4] =mg{h-r-rsin 9] (ii) Total kinetic energy of the ball = ; mv? + : mv? = = mv? 7 to mv? = mg(h-r-rsin 4] -. radial acceleration = g = 10s [4 -1-sin ‘| 7 On differentiating the above equation we have ua _ 108 6554. ro 7 one} dt ta. _ 5 Ax TTF cos } on? -. tangential acceleration a =— 58 cos > 3.43, On imparting an initial velocity v, to the ball shown in the figure it begins to move in a horizontal circle of radius R on the horizontal plane abcd as in Fig. 3.28. Assuming the coefficient of friction between the ball and the plane as »: determine the time required for the ball to come to Test. d Solution: Ja. =- ymgR; m= mass of the ball or mR . a =-pm R a M"s a I or ae Me 0 : frau =-ug joa a 1 b -¥ =-pg-t Fig. 3.28 or = Yo HE 3.44, A small object of mass m placed at the point O on the surface ofa smooth cylinder of radius R slides down along OA. At A it detaches from the surface and after travelling freely in air it strikes at horizontal plane at B (see Fig. 3.29). Find the length BC. Solution: At A we have v = 2gR (1 - cos 8) CIRCULAR MOTION AND GRAVITATION 131 2 and ae = mgcos® Se v? = Rgcos@ =2gR (1 —cos 6) or 3cos@=2 2 or cos @ = 3 Velocity at A = eee Horizontal component of this velocity v, = 2 Vertical component of this velocity v= \/—3— - si wi, et. 3 3 3 Point Ais at a height R + R cos @ above the horizontal plane. Time required to fall from this height is obtained by the equation. 10gR 40R 40R _40R xz st pee he 3g Vere 3g 40R 1,208 3g°°3 he oe = Fig. 3.29 te- 132 SOLVED PROBLEMS IN PHYSICS . 2 [agr for 2.162 Distance BC = 3 3° 3g xa +Rsin@ _ 4RV5 «2.162 | RV5 “a7 3 = R (0.7162 + 0.7453] =1461R 3.45. A smooth sphere of radius R is made to translate in a straight line witha constant acceleration a. A particle Akept on the top of the sphere is released from there at zero velocity with respect to the sphere (see Fig. 3.30). Find the speed of the particle with respect to the sphere as a function of angle 6. Solution: The sphere moves through a distance BC= Rsin 6 with anacceleration ahence velocity v, is given by vy = 2aR sin ® Now AB= R(1—cos 6) Velocity v, in the vertical direction is given by vz = 2gR (1 —cos 6) “P= vu; + v3 =2R [asin 0 + g-gcos 6] Fig. 3.30 1 or v= {2R(asin@ + g—g cosé)}? 3.46. A sleev A can slide freely along a smooth rod bent in the shape of a half circle of radius R as shown in Fig. 3.31a. The system is set in rotation with a constant angular velocity o about a vertical axis OO’. Find the angle 6 corresponding to the steady position of the sleeve. Solution: Normal force Nwill act along AB (see Fig. 3.31b). Nsin 0 = mo? Rsin 0 and Nos 0 = mg 78. . sing = mo® Rsin 0 cos 6 Fig. 3.314 Fig. 3.31b aa You have either reached a page that is unavailable for viewing or reached your viewing limit for this book. aa You have either reached a page that is unavailable for viewing or reached your viewing limit for this book. aa You have either reached a page that is unavailable for viewing or reached your viewing limit for this book. aa You have either reached a page that is unavailable for viewing or reached your viewing limit for this book. CIRCULAR MOTION AND GRAVITATION 137 Mass of the sphere which will cause the force M'=4nA ['Pdr lo =nArt emit nk Gravitational force 3.53. A hemispherical thin shell ofmass M and radius Rhas a small particle of mass m placed at its centre. Find the force of attraction between them. Solution: Consider an element of width Rd6 (see Fig. 3.36). Mass of the element = M + 2nR sin 0 - RdO 2nR Gravitational force between the particle GmM sin®- do and element = ——a Net gravitational force = oan sin cos6 dO Fig.g38 _GMm ~ 2k 3.54. A thin rod AB of mass M and length 2! and a particle of mass m is placed as shown in the Fig. 3.37. Calculate the gravitational force of attraction between the rod and the particle. yaxis B | x axis Solution: Consider an element of length dx situated at a distance x from the point Cin the AC portion of the rod. Fig. 3.37 Gravitational field due to this element at O = oe, al (a +x’) 138 SOLVED PROBLEMS IN PHYSICS Gravitational force between the element and particle _ GMm dx a (a +3") Net gravitational force along -ve side of x axis = aera 2I(a’ +x’) GMm = Da (L-cos 6) Net gravitational force along -ve side ofy axis = em eee al (@ +27) | Mim + sind ~ Qhea Similarly net gravitational force between BC portion of the rod and particle along +ve side of x axis = oe (1-cos $) Net gravitational force between BC portion of the rod and the particle along -ve side of y axis = GM Qa sino Total gravitational force along -ve side of y axis = Ff, = a (sin 6 + sin >) Total gravitational force along +ve side of x axis GMm = Sha (cos © —cos $) Result gravitational force between the rod AB and particle. _ GMm z 2 = Oba 9) +(cos @—cos $) GMm 3.55. A homogenous rod of mass m, is turned in the form of semi-circle of radius R as shown in Fig. 3.38. Calculate the force experienced by a particle of mass m, placed at the centre of the semi-circle. Solution: Mass per unit length of the wae m, semi-circle = — 1. Consider an element of length dL mass of the element = mal CIRCULAR MOTION AND GRAVITATION 139 Gravitational force between the element and the particle of mass m, is given by G ak Re Out ofthe twocomponents of the above force the effective component is given by Smt . = -sind G. in 8- Rde Net gravitational force = F=2 [2 a 2G-mm, (% =o cos0d® at _ 26mm, "aR? 3.56. A rod of length L is placed along x axis keeping its one end at a distance / from the origin. The mass per unit length varies with distance x by the relation 1 = k, + k, x* where k, and k, are positive constants. Find the force experienced by a particle of mass m placed at the origin. Solution: Consider an element of length dx situated at a distance x from the origin (see Fig. 3.39). Force between the particle and the element is given by i —{ J], es #— | —e—— 1 ——_- +#———_ x ——_ Fig. 3.39 . G-m-(k, +k,x*)-dx 2 aF x Mtoe be Net force F= far =G-m-k, i +G-m-k, fax ix t 1 1 =6-m-h, [b-oh ]+eem-a _ Gm-k,-L ~ U(l+ L) 3.57. A ring is made ofa thin wire of radius r. Find the force of attraction between the mass point of mass m placed along the axis of the ring of radius R at a distance x from its centre. +Gm-k,L Solution: Consider an element of length d/ on the circumference of the 140 SOLVED PROBLEMS IN PHYSICS circle of radius R. The mass of this element = nr-dl - p where pis the density of the material of the wire. The force of attraction between the element chosen and the point Gmnar'dlp (R +x") This force will have two components dF sin © and dF cos 9. From considerations of symmetry, dF sin © vanishes. Therefore the net effective force Fig. 3.40 is given by F= {dF cose = amiGtipx. "a (R+xe}P _ 2n'r’RoGmx (R422)! 3.58. A small particle of mass m' is placed at the axis of a circular wire of mass m and radius r at a small distance from the centre as shown in Fig. 3.41. Show that the particle will execute simple harmonic motion. Find the period of oscillation. Solution: The gravitational field at O at adistance x from the centre is given by mass is dF = G-m-x <7? E,= ———> (P+) Equation of motion of the particle of mass Fig. 3.41 m' for small values of x is given by i . G:mx-m' m'- acceleration =— — 5 Gm ,. acceleration = — a3 displacement. The acceleration is proportional to displacement hence the particle will execute simple harmonic motion. 3.59. Calculate the intensity of gravitational field at P due to an infinitely long straight wire of mass per unit length o. aa You have either reached a page that is unavailable for viewing or reached your viewing limit for this book. aa You have either reached a page that is unavailable for viewing or reached your viewing limit for this book. aa You have either reached a page that is unavailable for viewing or reached your viewing limit for this book. aa You have either reached a page that is unavailable for viewing or reached your viewing limit for this book. aa You have either reached a page that is unavailable for viewing or reached your viewing limit for this book. aa You have either reached a page that is unavailable for viewing or reached your viewing limit for this book. CIRCULAR MOTION AND GRAVITATION 147 Now wate 22% r 2a du _ secOtan® dea @u _ sec’6+secd-tan*6 x dy ~ 2a i sec*@+sec@-tan’@ secO (2) = mie [seeesseeomee 3 | Fig. 3.47 _ _ mh*u’-sec* 0 7 a 25,2 =— awe? a __ 8a?mh’ - r 3.68. A particle of mass m approaches earth from a large distance with an initial speed v,. Calculate (i) the impact parameter b of the particle so that it passes off tangentially to the earth surface (ii) the ecentricity of the orbit. Solution: Initial angular momentum, L, = mu,b 1 Initial energy = 2 mu, Angular momentum at B = mvR; v = velocity 1 Final energy = a me? - a ; R= radius of earth In view of conservation of angular momentum we have c vb m YER es 28m neaie - 1 1 GMm rme, 2 B cael = Sa 2 muy 2 mv? R i we 8 1 2 bt 2GM (/ 5 OF Ue = 1) pe - \ 1 2GM |? bealie et) Fig. 3.48 ot 2 -r[rsa(t) 0,-, {ot Uy R 1 yb? Bis perigee of the orbit at = Sine Gar 148 SOLVED PROBLEMS IN PHYSICS p yb v,) E tricity e = -l= Pegi] ccen' icity @ Tan G * ( } Since v, >v, “e>l The path is therefore hyperbola. 3.69. A space vehicle travels along a parabolic path with the earth at its focus. Calculate its total mechanical energy. Solution: When the vehicle is at the apex of the parabola, its total energy 2 w= oun ; r,s the distance between earth and vehicle f mu; For curved path we have ie = R, ;R, = radius of curvature. ‘ GMm _ my ; n &® my _ Ms, 1h wWw=—- =m |} 2 Ry *|2 Ry R yey . W=0 , ‘ig a 3.70. A particle of mass mis in a force field of the type F=— 7 e,, Where ais a positive constant, and e, is the unit vector of the particles position vector. The particle was placed at a point where the position vector is r, and the initial velocity v, perpendicular to r, was imparted to it. Find the trajectory of the particle. 1 Solution: Taking r = a the differential equation for the path of the particle in a central field is (2 au u 5 wt awe where h=76 = Potential energy V=- ffar = General equation of conic is p l "= Trecosd 1. Ecos pp =-au CIRCULAR MOTION AND GRAVITATION 149 1 a us Fs = Tan * 0088 »_ 2(E-V) duy From conservation of energy (% + mht do On putting the values we have 2 esinor + ( <-+Co0s0] = BE 22, ( 2, +080] m mit * mh? mh? a 2E C= + mth mh z ust = ae fs (eat cso} z 1 Eccentricity <= jE me and E= > mv2- Gi: @ 2 "% 2 2a For hyperbola mv? - 20 . Osince ¢>1 or %> sh nr fo 0 20 For parabola € =1 «. v,= J mr, . . 2a For ellipse e <1». vy, < i 3.71. A body of mass m is thrown vertically up with an initial velocity u from the north pole of the earth. To what height will the body ascend? Solution: Let us consider a body of mass m. Let it be thrown vertically up from the north pole N of the earth with an initial velocity u and let it ascend up to A such that AN = h (see Fig. 3.49). In view of conservation of energy, -GMm _{-GMm)_1 nie R,+h R. =e aa You have either reached a page that is unavailable for viewing or reached your viewing limit for this book. CIRCULAR MOTION AND GRAVITATION 151 (ii) Time period = = 3.73. Calculate the magnitude and the direction of the gravitational field at a point P on the perpendicular bisector of the equal masses separated by adistance 2/ as shown in the Fig. 3.51. For what value of r the field will be maximum? Solution: The gravitational field at a point P due to one mass is given by -_G:m (P +r) é Net gravitational field due to both the masses I = ey +r _ _2Gmr ae (P+rP This will be in the direction PO. For maxima or minima 1 3 +23 g =O=2Gm a SS " (P+) (F+ry 2Gm(P -2r’) = Fig. 3.54 ene ote “te oe . a . L With this value of r, = assumes a negative value hence for r= —= dr’ V2 the gravitational field is maximum. 3.74. Calculate the force of attraction on a particle of mass m placed at the vertex of a solid cone of mass M, height h and radius R as shown in Fig. 3.52a. Solution: Consider an element of thickness dy at a depth y from the vertex. The element will bein the form of disc. Upon this disc consider an element of radius x and x+ dx. Volume of the element = 22 xdxdy M 3 Mass of the element = TRA + 2nxdx - dy 152 SOLVED PROBLEMS IN PHYSICS [| ’ A-|y) Fig. 3.52a Fig. 3.52b Gravitational force between the element and particle of mass m _G-6M-m dy: xdx RH Oe) Effective gravitational force = SM dy ig cos © (see Rh (# +y¥) Fig. 3.52b) yanaG-6Mm — dy-xdx-y Net gravitational force due to the disc = 5 o Rh (e+y’) 6GMm Rh 6GMm h oR bate Net gravitational force between the cone and particle is given Fs far = ['8oMm ara wy 0 Rh Jh? +R _6GMm f hk - Rk fh? +R? 3.75. A solid hemisphere of mass Mand radius Rhas a particle of mass m placed at its centre. Calculate the force of attraction between them. Solution: Consider an element of thickness dy located at a height yabove the centre (see Fig. 3.53). The element is in the form of disc of radius a= JR’ -y’ . Upon this disc consider an element of radius x and x + dx. dF= + dy (1-cos a) aa You have either reached a page that is unavailable for viewing or reached your viewing limit for this book. aa You have either reached a page that is unavailable for viewing or reached your viewing limit for this book. aa You have either reached a page that is unavailable for viewing or reached your viewing limit for this book. aa You have either reached a page that is unavailable for viewing or reached your viewing limit for this book. aa You have either reached a page that is unavailable for viewing or reached your viewing limit for this book. aa You have either reached a page that is unavailable for viewing or reached your viewing limit for this book. aa You have either reached a page that is unavailable for viewing or reached your viewing limit for this book. 160 SOLVED PROBLEMS IN PHYSICS 4n Jap =- Gee > frar 4n 7? PEAG Pet et Re atr=R,p=0 4n FR acs Gast. Ge 3 2 p= 2G. pt Re-r Qn 9M? =o Teer [R?-r] _ 3GM?* 1- eC ~ 8nR* R 3.86. A chain oflength /and mass mis placed on the surface of asmooth solid sphere of radius R (R > !) with one end fixed at point O as shown in Fig. 3.61a. Find (i) the gravitational potential energy of the chain in reference to the centre of the sphere (ii) the velocity acquired by the chain after sliding through an angle 6 (iii) the tangential acceleration of the chain as it begins to slide. Solution: (i) Consider a small length dl of the chain. Fig. 3.60 Fig. 3.61a Fig. 3.61b di? = dx + dy or dL= Jdx? + dy" Inacircle x+y =F? xdx + ydy=0 x «dy=-— dx ” y aa You have either reached a page that is unavailable for viewing or reached your viewing limit for this book. aa You have either reached a page that is unavailable for viewing or reached your viewing limit for this book. aa You have either reached a page that is unavailable for viewing or reached your viewing limit for this book. aa You have either reached a page that is unavailable for viewing or reached your viewing limit for this book. aa You have either reached a page that is unavailable for viewing or reached your viewing limit for this book. aa You have either reached a page that is unavailable for viewing or reached your viewing limit for this book. aa You have either reached a page that is unavailable for viewing or reached your viewing limit for this book. aa You have either reached a page that is unavailable for viewing or reached your viewing limit for this book. aa You have either reached a page that is unavailable for viewing or reached your viewing limit for this book. aa You have either reached a page that is unavailable for viewing or reached your viewing limit for this book. aa You have either reached a page that is unavailable for viewing or reached your viewing limit for this book. aa You have either reached a page that is unavailable for viewing or reached your viewing limit for this book. aa You have either reached a page that is unavailable for viewing or reached your viewing limit for this book. aa You have either reached a page that is unavailable for viewing or reached your viewing limit for this book. aa You have either reached a page that is unavailable for viewing or reached your viewing limit for this book. aa You have either reached a page that is unavailable for viewing or reached your viewing limit for this book. aa You have either reached a page that is unavailable for viewing or reached your viewing limit for this book. aa You have either reached a page that is unavailable for viewing or reached your viewing limit for this book. aa You have either reached a page that is unavailable for viewing or reached your viewing limit for this book. aa You have either reached a page that is unavailable for viewing or reached your viewing limit for this book. aa You have either reached a page that is unavailable for viewing or reached your viewing limit for this book. aa You have either reached a page that is unavailable for viewing or reached your viewing limit for this book. aa You have either reached a page that is unavailable for viewing or reached your viewing limit for this book. aa You have either reached a page that is unavailable for viewing or reached your viewing limit for this book. aa You have either reached a page that is unavailable for viewing or reached your viewing limit for this book. aa You have either reached a page that is unavailable for viewing or reached your viewing limit for this book. aa You have either reached a page that is unavailable for viewing or reached your viewing limit for this book. aa You have either reached a page that is unavailable for viewing or reached your viewing limit for this book. aa You have either reached a page that is unavailable for viewing or reached your viewing limit for this book. aa You have either reached a page that is unavailable for viewing or reached your viewing limit for this book. aa You have either reached a page that is unavailable for viewing or reached your viewing limit for this book. aa You have either reached a page that is unavailable for viewing or reached your viewing limit for this book. aa You have either reached a page that is unavailable for viewing or reached your viewing limit for this book. aa You have either reached a page that is unavailable for viewing or reached your viewing limit for this book. aa You have either reached a page that is unavailable for viewing or reached your viewing limit for this book. aa You have either reached a page that is unavailable for viewing or reached your viewing limit for this book. aa You have either reached a page that is unavailable for viewing or reached your viewing limit for this book. aa You have either reached a page that is unavailable for viewing or reached your viewing limit for this book. aa You have either reached a page that is unavailable for viewing or reached your viewing limit for this book. aa You have either reached a page that is unavailable for viewing or reached your viewing limit for this book. aa You have either reached a page that is unavailable for viewing or reached your viewing limit for this book. aa You have either reached a page that is unavailable for viewing or reached your viewing limit for this book. aa You have either reached a page that is unavailable for viewing or reached your viewing limit for this book. aa You have either reached a page that is unavailable for viewing or reached your viewing limit for this book. aa You have either reached a page that is unavailable for viewing or reached your viewing limit for this book. aa You have either reached a page that is unavailable for viewing or reached your viewing limit for this book. aa You have either reached a page that is unavailable for viewing or reached your viewing limit for this book. aa You have either reached a page that is unavailable for viewing or reached your viewing limit for this book. aa You have either reached a page that is unavailable for viewing or reached your viewing limit for this book. aa You have either reached a page that is unavailable for viewing or reached your viewing limit for this book. aa You have either reached a page that is unavailable for viewing or reached your viewing limit for this book. aa You have either reached a page that is unavailable for viewing or reached your viewing limit for this book. aa You have either reached a page that is unavailable for viewing or reached your viewing limit for this book. aa You have either reached a page that is unavailable for viewing or reached your viewing limit for this book. aa You have either reached a page that is unavailable for viewing or reached your viewing limit for this book. aa You have either reached a page that is unavailable for viewing or reached your viewing limit for this book. aa You have either reached a page that is unavailable for viewing or reached your viewing limit for this book. aa You have either reached a page that is unavailable for viewing or reached your viewing limit for this book. aa You have either reached a page that is unavailable for viewing or reached your viewing limit for this book. aa You have either reached a page that is unavailable for viewing or reached your viewing limit for this book. aa You have either reached a page that is unavailable for viewing or reached your viewing limit for this book. aa You have either reached a page that is unavailable for viewing or reached your viewing limit for this book. aa You have either reached a page that is unavailable for viewing or reached your viewing limit for this book. aa You have either reached a page that is unavailable for viewing or reached your viewing limit for this book. aa You have either reached a page that is unavailable for viewing or reached your viewing limit for this book. aa You have either reached a page that is unavailable for viewing or reached your viewing limit for this book. aa You have either reached a page that is unavailable for viewing or reached your viewing limit for this book. aa You have either reached a page that is unavailable for viewing or reached your viewing limit for this book. aa You have either reached a page that is unavailable for viewing or reached your viewing limit for this book. aa You have either reached a page that is unavailable for viewing or reached your viewing limit for this book. aa You have either reached a page that is unavailable for viewing or reached your viewing limit for this book. aa You have either reached a page that is unavailable for viewing or reached your viewing limit for this book. aa You have either reached a page that is unavailable for viewing or reached your viewing limit for this book. aa You have either reached a page that is unavailable for viewing or reached your viewing limit for this book. aa You have either reached a page that is unavailable for viewing or reached your viewing limit for this book. aa You have either reached a page that is unavailable for viewing or reached your viewing limit for this book. aa You have either reached a page that is unavailable for viewing or reached your viewing limit for this book. aa You have either reached a page that is unavailable for viewing or reached your viewing limit for this book. aa You have either reached a page that is unavailable for viewing or reached your viewing limit for this book. aa You have either reached a page that is unavailable for viewing or reached your viewing limit for this book. aa You have either reached a page that is unavailable for viewing or reached your viewing limit for this book. aa You have either reached a page that is unavailable for viewing or reached your viewing limit for this book. aa You have either reached a page that is unavailable for viewing or reached your viewing limit for this book. aa You have either reached a page that is unavailable for viewing or reached your viewing limit for this book. aa You have either reached a page that is unavailable for viewing or reached your viewing limit for this book. aa You have either reached a page that is unavailable for viewing or reached your viewing limit for this book. aa You have either reached a page that is unavailable for viewing or reached your viewing limit for this book. aa You have either reached a page that is unavailable for viewing or reached your viewing limit for this book. aa You have either reached a page that is unavailable for viewing or reached your viewing limit for this book. aa You have either reached a page that is unavailable for viewing or reached your viewing limit for this book. aa You have either reached a page that is unavailable for viewing or reached your viewing limit for this book. aa You have either reached a page that is unavailable for viewing or reached your viewing limit for this book. aa You have either reached a page that is unavailable for viewing or reached your viewing limit for this book. aa You have either reached a page that is unavailable for viewing or reached your viewing limit for this book. aa You have either reached a page that is unavailable for viewing or reached your viewing limit for this book. aa You have either reached a page that is unavailable for viewing or reached your viewing limit for this book. aa You have either reached a page that is unavailable for viewing or reached your viewing limit for this book. aa You have either reached a page that is unavailable for viewing or reached your viewing limit for this book. aa You have either reached a page that is unavailable for viewing or reached your viewing limit for this book. aa You have either reached a page that is unavailable for viewing or reached your viewing limit for this book. aa You have either reached a page that is unavailable for viewing or reached your viewing limit for this book. aa You have either reached a page that is unavailable for viewing or reached your viewing limit for this book. aa You have either reached a page that is unavailable for viewing or reached your viewing limit for this book. aa You have either reached a page that is unavailable for viewing or reached your viewing limit for this book. aa You have either reached a page that is unavailable for viewing or reached your viewing limit for this book. aa You have either reached a page that is unavailable for viewing or reached your viewing limit for this book. aa You have either reached a page that is unavailable for viewing or reached your viewing limit for this book. aa You have either reached a page that is unavailable for viewing or reached your viewing limit for this book. aa You have either reached a page that is unavailable for viewing or reached your viewing limit for this book. aa You have either reached a page that is unavailable for viewing or reached your viewing limit for this book. aa You have either reached a page that is unavailable for viewing or reached your viewing limit for this book. aa You have either reached a page that is unavailable for viewing or reached your viewing limit for this book. aa You have either reached a page that is unavailable for viewing or reached your viewing limit for this book. aa You have either reached a page that is unavailable for viewing or reached your viewing limit for this book. aa You have either reached a page that is unavailable for viewing or reached your viewing limit for this book. aa You have either reached a page that is unavailable for viewing or reached your viewing limit for this book. aa You have either reached a page that is unavailable for viewing or reached your viewing limit for this book. aa You have either reached a page that is unavailable for viewing or reached your viewing limit for this book. aa You have either reached a page that is unavailable for viewing or reached your viewing limit for this book. aa You have either reached a page that is unavailable for viewing or reached your viewing limit for this book. aa You have either reached a page that is unavailable for viewing or reached your viewing limit for this book. aa You have either reached a page that is unavailable for viewing or reached your viewing limit for this book. aa You have either reached a page that is unavailable for viewing or reached your viewing limit for this book. aa You have either reached a page that is unavailable for viewing or reached your viewing limit for this book. aa You have either reached a page that is unavailable for viewing or reached your viewing limit for this book. aa You have either reached a page that is unavailable for viewing or reached your viewing limit for this book. aa You have either reached a page that is unavailable for viewing or reached your viewing limit for this book. aa You have either reached a page that is unavailable for viewing or reached your viewing limit for this book. aa You have either reached a page that is unavailable for viewing or reached your viewing limit for this book. aa You have either reached a page that is unavailable for viewing or reached your viewing limit for this book. aa You have either reached a page that is unavailable for viewing or reached your viewing limit for this book. aa You have either reached a page that is unavailable for viewing or reached your viewing limit for this book. aa You have either reached a page that is unavailable for viewing or reached your viewing limit for this book. aa You have either reached a page that is unavailable for viewing or reached your viewing limit for this book. aa You have either reached a page that is unavailable for viewing or reached your viewing limit for this book. aa You have either reached a page that is unavailable for viewing or reached your viewing limit for this book. aa You have either reached a page that is unavailable for viewing or reached your viewing limit for this book. aa You have either reached a page that is unavailable for viewing or reached your viewing limit for this book. aa You have either reached a page that is unavailable for viewing or reached your viewing limit for this book. aa You have either reached a page that is unavailable for viewing or reached your viewing limit for this book. aa You have either reached a page that is unavailable for viewing or reached your viewing limit for this book. aa You have either reached a page that is unavailable for viewing or reached your viewing limit for this book. aa You have either reached a page that is unavailable for viewing or reached your viewing limit for this book. aa You have either reached a page that is unavailable for viewing or reached your viewing limit for this book. aa You have either reached a page that is unavailable for viewing or reached your viewing limit for this book. aa You have either reached a page that is unavailable for viewing or reached your viewing limit for this book. aa You have either reached a page that is unavailable for viewing or reached your viewing limit for this book. aa You have either reached a page that is unavailable for viewing or reached your viewing limit for this book. aa You have either reached a page that is unavailable for viewing or reached your viewing limit for this book. aa You have either reached a page that is unavailable for viewing or reached your viewing limit for this book. aa You have either reached a page that is unavailable for viewing or reached your viewing limit for this book. aa You have either reached a page that is unavailable for viewing or reached your viewing limit for this book. aa You have either reached a page that is unavailable for viewing or reached your viewing limit for this book. aa You have either reached a page that is unavailable for viewing or reached your viewing limit for this book. aa You have either reached a page that is unavailable for viewing or reached your viewing limit for this book. aa You have either reached a page that is unavailable for viewing or reached your viewing limit for this book. aa You have either reached a page that is unavailable for viewing or reached your viewing limit for this book. aa You have either reached a page that is unavailable for viewing or reached your viewing limit for this book. aa You have either reached a page that is unavailable for viewing or reached your viewing limit for this book. aa You have either reached a page that is unavailable for viewing or reached your viewing limit for this book. aa You have either reached a page that is unavailable for viewing or reached your viewing limit for this book. aa You have either reached a page that is unavailable for viewing or reached your viewing limit for this book. aa You have either reached a page that is unavailable for viewing or reached your viewing limit for this book. aa You have either reached a page that is unavailable for viewing or reached your viewing limit for this book. aa You have either reached a page that is unavailable for viewing or reached your viewing limit for this book. aa You have either reached a page that is unavailable for viewing or reached your viewing limit for this book. aa You have either reached a page that is unavailable for viewing or reached your viewing limit for this book. aa You have either reached a page that is unavailable for viewing or reached your viewing limit for this book. aa You have either reached a page that is unavailable for viewing or reached your viewing limit for this book. aa You have either reached a page that is unavailable for viewing or reached your viewing limit for this book. aa You have either reached a page that is unavailable for viewing or reached your viewing limit for this book. aa You have either reached a page that is unavailable for viewing or reached your viewing limit for this book. aa You have either reached a page that is unavailable for viewing or reached your viewing limit for this book. aa You have either reached a page that is unavailable for viewing or reached your viewing limit for this book. aa You have either reached a page that is unavailable for viewing or reached your viewing limit for this book. aa You have either reached a page that is unavailable for viewing or reached your viewing limit for this book. aa You have either reached a page that is unavailable for viewing or reached your viewing limit for this book. aa You have either reached a page that is unavailable for viewing or reached your viewing limit for this book. ELASTICITY 365 6. Find (i) the torque developed (ii) the amount of work done (iii) compare the modulus of rigidity of a hollow and solid shaft. Solution: @ BB’ = x 0=1 4 (see Fig. 6.21) or $= 2» _ Shearing stressf — fl ~ 6 ~ x0 f= = “ l Consider a shell of radius xand x + dx Area of the shell = 2x xdx Shearing force tangential to the surface of the cylinder = a -2nxdx Torque acting on the cylinder Fig. 6.21 dt= ome +xXPdx-x Net torque = fare = ae i xdx _ mo (re “) - 21 ‘oa +_,4)92 ii) Workdone = [rd0 = pe a. ae - mle nn(r3 - 1) 21 (ii) Torsional sigh C= for hollow cylinder © far solid cylinder S (f-8)(e+n) aoe If masses of both the cylinders are same then a(7-1 lp = xP lp Rar Co aw +en Now c= = 2 =l+ = uG@ oC 7 SimpLE Harmonic Motion 7.1. Show that a particle moving on the circumference ofa circle executes simple harmonic motion. Solution: Let a particle be at A (see Fig. 7.1). It starts moving on the circumference of the circle with an angular velocity and reaches B after time tf. Its displacements along the x and y axes is given by x=acosot y axis y=asinot ‘ 1 where ais the radius of the circle. The acceleration along the x and y-axes is given by ax ad Ge =r oxand FF =-oty x axis Fig. 7.1 then = a@cosat=o ay dt ar ec edt la — x? SIMPLE HARMONIC MOTION 367 Integrating the above equation within the limits given, the required time is as follows: 4 dx Qn f = — |dt lees! or sin” x wth 4 3 or sin 1-sin* ; = a nt an otis enelep or 26° 3 t 2n Ba hhey or 3 3 or t=0.5s 7.3. Two linear simple harmonic motions of different amplitudes and same frequency are imposed on a particle along the x and y axes respectively. Find out the resultant path followed by the particle if the initial phase difference between them is 0, x and n/2 respectively. Solution: Let the equation of motion of a particle along x and yaxes be x =asinot y = bsin (ot+ 9) where $is the initial phase. Now x = sinwtcosd+cosotsing ye. or a= Y_X cosy) i-=cos¢| = or ( ba ‘) 2 42 2. or a + a = = cosd = (i) When$=0we have y eo = =Oory= 2 x, i.e., the particle will describe a straight line in the first and third quadrants. (ii) When $=, we have y iad - =Oory= 2 x, i.e., the particle will describe a straight line in the second and fourth quadrants. 368 SOLVED PROBLEMS IN PHYSICS (iii) When $ = 2/2, we have 2 2 ate =1 i.e. the particle will describe an ellipse. ¢ When a= b, the above equation is transformed into the equation of the circle x° + = a 7.4. Two linear simple harmonic motions of equal amplitudes and frequencies w and 2@ are imposed on a particle along the axes of x and y respectively. If the initial phase difference between them is r/2, then find the resultant path followed by the particle. (Roorkee 1985) Solution: Let the equation of the particle along the x and y axes be x =acoswtandy=acos (20¢ + =) Now y =-asin2ot = -2asinetcos ot xox @a or y =42 (-3) 7.5. A particle executes a simple harmonic motion with time period T, = 3s under a constraining force F,, with time period T, = 4s under a similar force F,, What will be its time period when the two forces acts together? Solution: Let m be the mass of the particle. Suppose the first force F, causes an acceleration a, and the second force F, and acceleration a, in it. Also, assume that the sum of these forces produces an acceleration a in the particle. Then, according to Newton's second law =-2a Jl- FE E, F+kh a, = ies ay anda= 1 @=Q,+4, Since the particle executes simple harmonic motion, @, =- fx, 4,=- 03x anda=- ox where ©, = BR = 2r andes 2s and xis the displacement. 1 pO TE T Now @? = of +} o 2etyt Pr TT = nh _ "eee SIMPLE HARMONIC MOTION 369 On substituting the values of T, and T,, we have 3x4 T= 3° +4 7.6. A block is resting on a piston which is moving vertically in simple harmonic motion of period 1.0 s. At what amplitude of motion will the block and piston separate? What is the maximum velocity of the piston at this amplitude? (Roorkee 1985) Solution: Let us assume that the piston holding the block is executing simple harmonic motion along the y-axis. The equation of motion is given by y =asinot 2 = velocity =a cosat @y and wa 2 = ion=-ay, | 22 ae = acceleration =- ay, ( dt), The maximum value of acceleration at which the block separates should be equal to g, i.e., oa=g = .r2 8-88 _. oe ae "ae 7 Gaia 7 0748m The maximum velocity of the particle is equal to oa= = +a =2x3.14%0.248=1.56 m/s 7.7, Aparticle starting from the origin executes simple harmonic motion. along xaxis. Its velocity at any instant tis given by v, = 22 cos xt/2 cm/sec. Calculate the total distance covered by the particle in time t = 4.5 sec. Solution: ant T 2 “T= 4sec ax t Y= Fp TAA St x=2x sin Zt n 2 or x= Msin rom Distance travelled in 4 sec = 4 x 14 = 56cm nn Distance travelled in remaining 0.5 sec = x= 14 sin 2% 0.5 14 = +t 2799 =9898em E 2 370 SOLVED PROBLEMS IN PHYSICS ~. Total distance covered = 56 + 9.9 =65.9cm = 0.659 m. 7.8. A particle participates simultaneously in two simple harmonic a ; 35 mt 35 oscillations in the same direction; x, = — cos = cmand x,= — cos xtcm. T od 2 Calculate the maximum velocity of the particle. Solution: dx, nm 35 on dx, 3. A gin ts See ax.- t at 2 gh Grates sina vse SI sin Zt 4sin St . cos] 2 2 2 2 ut Put z =6 vs = [sin 0 + 4 sin 6 - cos 6] dq _,__ 35 ‘i = 70 =0O=- 3 [cos 6 + 4 cos? 6-4 sin? 6] cos 6 + 4 cos? 8-4 (1-cos’ 6) =0 8cos?8+cos8-4=0 -14J/1+128 16 e630 = = 0.6474 sin 6 = 0.762 35 Vaggx = 10.762 +4 x 0.762 x 0.6474) = 47.86 cm/sec 7.9. Aspring of force constant K suspended from the ceiling of an elevator contains a ball of mass m at its other end. At an instant f= 0 the elevator is at rest and at the origin. It starts going up with an acceleration a = at. Where « is constant. Find the law of motion of the ball relative to the elevator. Solution: The ball will execute simple harmonic motion with frequency o= J— m Let the acceleration be given by ay 5 a Ao’ sin ot On integration dy aust —Awcos ot +C 372 SOLVED PROBLEMS IN PHYSICS 7.11. A plank having a body of mass m on it starts going up and its displacement from the initial position is given by the relation. y=a(1-cos wf) where ais constant. Find (i) the force exerted by the body on the plank at an instant t (ii) the amplitude of oscillation of the plank at which the body starts falling behind the plank (iii) the amplitude of oscillation of the plank at which the body jumps upto a height h with respect to its initial position. Solution: Wi) y =a(1-cos wt) at at v =aosinot ad’y = sae a= aw’ cos ot Force due to motion = ma w’ cos wt Force exerted by the body = mg + ma w? cos at 2 =mg [se : covat| (ii) =4,,,° 0? (iii), Maximum velocity v = Ao; A= amplitude Mo? = ¥ = 2gh 2 A= 7 Maximum amplitude = A - a, = 2gh ae o 7.12. A ball of mass m is suspended by a massless spring of force constant K. The ball is set free without any push. Find (i) the displacement of the ball from its equilibrium position as a function of time (ii) the maximum value of the tension produced in the spring. Solution: (i) The equation of motion of the ball is given by md?x “ae 7 K oO= J— SIMPLE HARMONIC MOTION 373 Let the displacement of ball be represented by the equation y = A(1-cos of) ay r = =A t en osina! d’y ; qe? = Avtcosot a att=0 a 7s “g = Aw «fn or A= wey ms K (1-cos wf) (ii) T-mg =ma T =m(g+a) Toy = 2ME 7.13. A particle of mass m having an initial displacement r, along x axis and initial velocity v, along y axis moves under the action of a force F=-am (ix+ jy) where aisa positive constant. Find the path of the particle. Solution: The acceleration along x axis is given by Li ae” Let x = x, Cos wr be the solution of the equation -ax ax 5 cos Va-t att=0, x =r. X=7, 7% = 7, cos Va-t The acceleration along y axis is given by dy are ~~ Let y=, sin wt be the solution of the equation dy then at =y,0 cost 374 SOLVED PROBLEMS IN PHYSICS wy. or y= Je sin(va . t) The path of particle is given by (2) = 7.14. A particle of mass m is executing simple harmonic motion about its equilibrium position with an amplitude a. During oscillation when itis at a distance x from the equilibrium position the particle receives a blowin the direction of motion and as a consequence of that its velocity becomes 1.5 times the velocity at that point. Find the new amplitude of the particle. Solution: The velocity in simple harmonic motion is given by v= aya -x As a result of receiving the blow the velocity becomes 1.5v. or 1.5v = 0. yA -2x ; Ais the new amplitude of motion. or 15.Ja-2 = Jae- x? 2.25 @ 2.257 = A?- or A= J2.25a’-1.25x" 7.15. A particle of mass m performs simple harmonic motion with an amplitude a. The angular frequency of oscillation is @. In the state of oscillation when it is at a distance x from the mean position an impulse Jis given to the particle in the same direction. Find the new amplitude of oscillation. Solution: The velocity of a particle executing simple harmonic motion is given by v Now T= my’ SIMPLE HARMONIC MOTION 375 7.16. A particle of mass m starts moving from a point where x= x, with an initial velocity u along a straight line under the action ofan attraction force mo2x toward a point O lying on the line where x is the distance from O. Find the displacement. Solution: Here acceleration and displacement are inter related by the equation Acceleration =- @?+ x This is the case of simple harmonic motion. Let the solution be x =Asinot+ Bcosot ax att=0, x =x, and ae =u(given) Now B=x, & =Aocosot-Bcossin ot u=Ao ee ao xX =X,Cosat+ “ sin ot 7.17. Describe the simple harmonic motion when the displacements are 3 3 0, Bemana ~yematt, =0, t,=2 secand t,=3 sec. Solution: Let the equation of the simple harmonic motion be x = asin (t+ 9) o = asin (+9) O=-@ e = asin (20 + 9) =asino 3 Similarly 3 = asin 3a +) =asin2o 3 5 5 =2cos@-asine 3 = 2cosw-—— 3 “COS@ = -—— 2 5x or o=— 6 376 SOLVED PROBLEMS IN PHYSICS 5 Now = = asin = a= V3 cm x = ¥3 sin (%--o} 5 = ¥3 sin - (t-1) cm 7.18. A particle moves simple harmonically along a straight line. It starts from equilibrium without any initial velocity and travells a distance I, in the first second and I, in the second second in the same direction. Find the amplitude of oscillation. Solution: Let the equation of motion of the particle be x = @(1-Cos of); a= amplitude In view of the conditions given 1, =a(1-coso) 1, +1, = a(1-cos 20) =2asin’o (hy «eh wt a 2a ehh bth oy a a 2a Fo 3he+h _ a 27° 2h or a= 3h-4 7.19. A particle oscillates simple harmonically along x axis. At times ¢, 2t and 3titis located at a distance a, band crespectively from the equilibrium position. Find the period of oscillation. Solution: In the light of equation x = A sin wf we have a =Asinot b = Asin2ot c= Asin30t Now a+c =Al[sinat+sin3o/] = 2Asin 2otcos ot = 2bcosat c@Pin coe: Cee <. ot = cos (= SIMPLE HARMONIC MOTION 377 2nt cos! {44+£ 2b 7.20. Aparticle oscillating simple harmonically has velocities v,, v, and v, respectively when it is at a distance of x,, x, and x, from the equilibrium position. Find its period of oscillation. or T= Solution: vi =o? (a’ 21), vf =0* (a ~ x2) and v3 =0? (a’ ~ x3) yo X, x, x] = 0? (a? - 27) (x,-x,) +0? (a” - x3) (x,-x) 111 +? (a’ ~ x4) (x,-x,) =o?[a* (x, —x,)+@? (x,-x,) +a? (x,-x,)- x7Xx, $x¢x, — OX, +45%, — 54, +252, | = w? [(x, — x) (x,-x,) &,- ,)] (41-= %)(% — 4 )(% = %) yy Us US x, x % 1o.iid 7.21. A particle of mass m is executing simple harmonic motion along x axis under the action of a force F = — kx with a period of 16 sec. In the course of motion it crosses the equilibrium position at r= 2 sec and acquire a velocity of 4 m/sec at t= 4 sec. Find (1) the equation of motion (ii) the amplitude of oscillation. Solution: m- acceleration =— kx k ~@ = m Let the equation of motion be x= a sin o(t-2) where a= amplitude of motion. Now ae = @@ COS @ (f-2) at 4 =a@cos2o oe @ Cos 20 378 SOLVED PROBLEMS IN PHYSICS T 1 8 __4 oor 8 2 _ 32V2 ~ cd x = 32v2 sin = (t-2) Tt 8 7.22. A particle of mass m is located in a unidimensional potential where the potential energy of the particle depends on the coordinate x as U(x) a_b a9 Find the period of small ocillations that the particle executes about the equilibrium position. Solution: The particle executes motion about the position of minimum energy. BU a 2 re) orca ao x ob a b ¢ Now ves Toa! p=displacement (F) (Fe) . aU F = mx acceleration =— —— dp 2a “ae ( °) Bb mxacceleration = “ “acceleration = ama - displacement o=2t 2 _ - T 2a-J2ma or Ts Ane ma 380 SOLVED PROBLEMS IN PHYSICS . 1 opal Average value of potential energy = (U,,,) = Sa lig KA “6 Kx’dx Total energy of the particle = > KA -. Average value of kinetic energy = (K-E,,) 2 ot [tl k(at-e)ar= A2 3 (ii) Uw) == hs 1 KAP costot- dt =i 0 "(1+ cos 2ot) dt KA? “4 (K-E,,) = 7 fg an ot-dt 2 — wr 7 (1—cos2wt)dt =A a 4 7.25. A weightless wooden stick whose one end is fixed at O is kept horizontal (Fig. 7.2). Am=0.5 kgload is applied at the other end produces adepression of = 1 cm. The stick is further depressed a little and released. Find the period of oscillation. Solution: Let c be the twisting couple per unit depression then in equilibrium, we have c-8= mel; 1=length of the stick Let the stick be further depressed by y then torque is given by mgl-c(&+y)=-cy ° The equation of motion is given by \ 8 \ a *) = rR. cyemn | 2 or mi 52 = m d’y --By Fig. 7.2 dt? 6 SIMPLE HARMONIC MOTION 381 “Ts an, g On substituting the values of gand 5 we have T = 2x3.14- _ 2x3.14 ~ 31.30 = 0.2 sec 7.26. A point particle of mass 0.1 kg is executing simple harmonic motion of amplitude of 0.1m. When the particle passes through the mean position, its kinetic energy is 8 x 10° Joule. Obtain the equation of motion of this particle if the initial phase of oscillation is 45°. (Roorkee 1991) Solution: Let the equation of simple harmonic motion be y=asin (at +) dy Velocity ye 749.008 (r+) When the particle passes through the mean position its velocity is maximum and is equal to oa. L Now 2 m-o? a@=8x 10") ; x0.1x@'x0.17=8x 10% @=t4 ~. Required equation is y =0.1 sin (+ 4t+ 7/4) meter 7.27. Acylindrical piece of wood of mass m and area of cross-section Ais floating erect in a liquid density p (see Fig. 7.3). It is pressed down slightly and then released. Show that its motion is simple harmonic, and calculate the time period of this motion. Solution: Let the cylinder be pressed vertically through a depth y. The volume of liquid displaced by it is given by V=Ay Upward thrust = weight of liquid displaced = Apgy This force is acting vertically upward, i.e., opposite to the displacementy. The equation of motion of the cylinder is given by Fig. 7.3 382 SOLVED PROBLEMS IN PHYSICS fy: m Ct =-Apgy a Ap or Gre i is proportional to the displacement y. The motion cf cylinder is therefore simple harmonic. A, Now o= aes m m ifa2n ‘Apg 7.28. The graduated cylindrical stem ofa lactometer has an area of cross- section A = 2 x 10° m?, Its mass m including the mass of the lead shots confined ina spherical space at its bottomis 100 g. Itis immersed in water of density p= 10°kg/m* to at a slightly greater depth than actually needed for equilibrium, and then released. Show that it will execute simple harmonic motion. Calculate its time period. Solution: Let the lactometer be pressed to a depth greater than that required for equilibrium in the vertical direction. Volume of water displaced = Ay. Upward thrust=Apgy This force is acting vertically upward, i.e., opposite to the displacement ¥ The equation of motion of the lactometer is given by This acceleration 2 ay maz =~Apsy dy Apg or a om” i The acceleration is proportional to the 1 Us displacement y. The motion is therefore simple r harmonic. L 2 [A © Now @= = = Ps Fig. 7.4 T = 2n | 2 T22n Pe Apg On substituting the values of the various terms occurring in the above equation, we have 384 SOLVED PROBLEMS IN PHYSICS 7.30. Two non-viscous, incompressible and immiscible liquids of densities p and 1.5 p are poured into two limbs of a circular tube of radius R and small cross-section kept fixed in a vertical plane as shown in the Fig. 7.6. Each liquid occupies one fourth the circumference of the tube. (a) Find the angle 6 that the radius vector to the interface make with the vertical in equilibrium position. (b) If the whole liquid is given a small displacement from the equilibrium position, show that the resulting oscillations are simple harmonic. Find the time period of these oscillations. (IT 1991) DS Fig. 7 Solution: (a) In equilibrium we have Ping = pph,8+ Phage 1.5pR(1-sin 6) = pR(sin 6 + cos 6) + 1.5 pR(1-cos6) or 2.5 sin® = 0.5cos® @ =tan? ) =11.3° 5 i (b) Displacement of the liquid is given by y=Ro Change in height of liquid column = h = R [sin (6 + $) — sin 6] h=R {[sin® cos 9-cos6 sin $] — sin 6) =Rcos 0; herecos$=1landsin d= Restoring force =-2.5p ARgcos6- ? ; A =area of cross-section =-2.5pAgcos0-y 2.5pAgcos 6 = k (force constant) F = mxacceleration =—ky o= at “Nm T m T = 2x,|—— % 2.5p Ag cos m=A. ae (L5p+pl=1.25nRp mR = 2x J-—— r * 3g coso SIMPLE HARMONIC MOTION 385 cos 11.3° = 0.98 x 7.31. One end of along metallic wire of length L is tied to the ceiling. The other end is tied to a massless spring of spring constant K. A mass m hangs freely from the free end of the spring. The area of cross-section and the Young's modulus of the wire are A and Yrespectively. Find the time period with which mass m will oscillate if it is slightly pulled down and released. (IIT 1993) Solution: F= Ky Where K_ is the total force constant and yis total elongation YEN +2 y, = elongation of the spring y, = elongation of the wire F HF K ~ HE AY, ey = Ze “w= ay Fm Y= xk * ay - (“2E)- AKY AKY Hs (4) jerationn = .{A8¥ mxaccelerationn = — | 7g y = 2m_ | AKY_ °* 'T \m(AY+KL) m (AY + KL) = 2nj—“— T= 2n 7.32. Two identical balls A and B each of mass 0.1 kg are attached to two identical massless springs. The spring-mass system is constrained to move inside a rigid smooth pipe bent in the form of a circle as shown in Fig. 7.7. The pipe is fixed in a horizontal plane. The centres of the balls can move in a circle of radius 0.06 m. Each spring has a natural length of 0.06 xm. and force constant 0.1 N/m. Initially both the balls are displaced by an 386 SOLVED PROBLEMS IN PHYSICS angle 0 = n/6 radiation with respect to diameter PQ of the circle and released from rest. (a) Calculate the frequency of oscillation of the Ball B. (b) What is the total energy of the system? (c) Find the speed of ball A when A and B are at the two ends of the diameter PQ. (IT 1993) Solution: (a) The reduced mass m of the system is given by 0.1x 0.1 m= reo OOo ks Combined force constant K=K,+K,=0.1+0.1=0.2 Frequency of oscillation 1 {kK ae 2n\m 1 j0.02 1 fae Hy 2nV0.05 1 1 1 (b) B= > Ky¥+> Ky where y= 2R0=2Rr/6 = 2 x 0.06 x 2/6 =0.02 7m 1 Energy E= > (0.1 +0.1] x (0.021)? =0.1 x (0.02 2)? =3,94x104J (©) > mv? + ; mv? = 3.94x 104 mv? = 3.94x 104 v= jaa x107 0.1 = 6.277 x 107 m/s 7.33. A solid cylinder of mass M is attached to a horizontal massless spring of force constant Kso as to roll without slipping along a horizontal surface as shown in the Fig. 7.8. The spring is stretched by x and released show that the centre of mass of the cylinder executes simple harmonic : . 3M motion of period T= 2x OK’ Solution: Total energy = $M 510+ 5K = Constant Fla 72 SIMPLE HARMONIC MOTION 387 1 MPa? = a Mv’; 3 1 gee lige t.t 4 Mv + 5 Ke? = constant; 5 fo*= 5 3 r= radius of the cylinder On differentiation 3 wy. & --keev 2 dt dv 2K Ow. ion = —-=-=—: ement acceleration dt 3M displacemé Hence the centre of mass will execute S.H.M. _ 2m. PK = 73M 3M “T= 2n OK 7.34. An ideal gas is enclosed ina vertical cylindrical container and supports a freely moving piston of mass M. The piston and the cylinder have the same area of cross-section A. Atmospheric pressure is P and when the piston is in equilibrium, the volume of the gas is V. The piston is now displaced slightly from the equilibrium position and left. Assuming that the system is completely isolated from its surroundings, show that the piston executes simple harmonic motion and find the frequency of oscillation. (IT 1981) Solution: P and V are, respectively, the pressure and volume of the gas enclosed in the cylinder-piston system under conditions of equilibrium (Fig. 7.9). Let the piston be pressed through a depth y. This will cause an increase of pressure and decrease in volume. Let the new pressure and volume P+ dP and V-dV respectively. The process is adiabatic since the system is completely isolated. Inan adiabatic process, PV'=constant On differentiating the above, we have aP.Vi+yP-V"-!.dv=0 y yP yP =--Lav=-L.a. or dP V V y Restoring force = F=dP-A 2 - iP i Fig. 7.9 wFe-Taty g @y Pp a? ~~ uv? 390 SOLVED PROBLEMS IN PHYSICS 7.37. Acylinder of length 2! enclosed at both ends contains a gas anda piston of mass mand area of cross-section A at its middle (Fig. 7.12). The piston divides the cylinder into two compartments having equal pressure Pand volume V. The piston is slightly displaced to the left suddenly and released. Show that it will execute simple harmonic oscillation. Find the frequency of oscillation assuming the processes in the gas to be an adiabatic. Solution: Volume of the gas V=A-/ Pressure of the gas = P On displacing the piston to the left horizontally through a distance x, let P, and P, be the pressures in the first and second y y y y y y H y y H y Z compartment. As the process is an adiabatic, hon 1:82 we have P, ((1—x) A}! = P, (1+ x) Al! = P(A)" or P, (I- x)" = P, (l+x)"= PLY ry (iy Force Facting on the piston = (P, - P,) A= PA (4) = (4) =PA [a-xy" ~(1+axnty] 22? 144 This force will bring the piston to its equilibrium position. The equation of motion of the piston is given by ax A z The acceleration re is proportional to the displacement x since 2 ve is constant. Therefore the piston will execute simple harmonic motion. 2PyA? Now a= mV 2Py n= On \mv SIMPLE HARMONIC MOTION 391 7.38. A point of mass M is suspended by a weightless wire of length L and area of cross-section A. If Yis the Young’s modulus of the material of the wire, then determine the frequency of simple harmonic motion of the mass in the vertical direction. Solution: The suspended mass will elongate the wire by a length J. According to Hooke's law, = Mg vol or Mg= YAl The weight Mg will be equal to the tension in the wire. Therefore YAl T = Mg= — 8 L On pulling the mass M vertically through a distance y, the tension will increase. The tension at that instant is given by AY T, = 74 +y) r AY Net force acting on the mass = T- T, =— -Y The equation of motion of the mass is given by dy AY Mag =: 7 “2 T° a MY d’y The acceleration “ae. 'S proportional to displacement y. Therefore the mass will execute simple harmonic motion. YA Now ©= or n= 7.39. Show that a particle dropped into a tunnel dug along the diameter of the earth (see Fig. 7.13) will execute simple harmonic motion. Find its time period. Solution: Let a particle of mass m be dropped through the tunnel and let it be at a distance y from the centre of the earth. For the purpose of gravitation only the mass of the unshaded portion of the earth will be 392 SOLVED PROBLEMS IN PHYSICS effective. The mas of this portion is M'= $n y+ p where p is the mean density of the earth. The force of attraction between m t and M’ is given by y F=- GMa (the force is directed towards the centre of the earth) or Fe-G-Snpmy The equation of motion of the particle is given by Fig. 7.13 a@y 4 map =-Ga mpm-y d’y 4 or wr -G 3 T™py ay The acceleration a is proportional to the displacement ysince (4/3) mp Gis constant. Therefore the particle will execute simple harmonic motion. an [Exp Now OFF 3 3x or T pG On substituting the mean density of earth p = 5.51 x 103 kg/m} and G=6.67 x 107! Nm?/kg? in the above, T= 84.2 min. 7.40. Show that a particle dropped into a tunnel AB dug through the earth (Fig. 7.14) will execute simple harmonic motion. Find its time period. Solution: Let m be the mass of particle and O be the centre of the earth. Force acting on the particle towards the centre of the earth in mg. Due to the force mg cos 0 the particle will move towards B. In the course of motion its velocity will go on increasing and it will have the highest value at C. As it moves from C towards B its velocity will continuously decrease and at B its velocity will be zero. At this stage the component force mg cos 6 will act in the direction BA and the particle will start moving up. SIMPLE HARMONIC MOTION 393 Thus restoring force = - mg cos 0 =— ie ;.R,=radius of the earth wake The equation of motion of the particle is or =- = ‘'y a . or t =- - “y a ’ y, ‘ r The acceleration 772 is proportional to the displacement. Therefore the particle will execute simple harmonic motion. 2n_ fg Now o=re R or T =2n= Re & On substituting R, = 64 x 10° m; T= 84.4 min 7.41. Asmail rectangular board is placed horizontally on two cylinders spinning about their own axes in the opposite sense (Fig. 7.15a). The axes are separated by a distance ! = 30 cm. The coefficient of friction between the board and cylinder is k= 0.15. Show that the board will perform simple harmonic motion. Find its time period. I mg Fig. 7.158 Solution: Let P, and P, be the lel/2 —xele— 1/2 4x pressures of the board on the respective cylinders (Fig. 7.15b). F, F, The friction forces acting on the board are given by mg F,=kP, and F,=k-P, On displacing the board to the left through a small distance x, the Fig. 7.15b two pressures P, and P, will be represented as (1/2 +x) mg P,= —s and P, = (1/2 - x) mg I where mg = weight of the board.

You might also like